MBE Contracts Questions

Pataasin ang iyong marka sa homework at exams ngayon gamit ang Quizwiz!

A microbrewery and a farming operation entered into a written contract for the sale of barley. The microbrewery agreed to buy from the farm "all the barley that the microbrewery requires" in its manufacture of beer, for a period of five years, at a mutually agreed-upon price. Under the contract, the microbrewery would place its orders on the first of each month and the barley would be delivered within five business days. For the first two years of the contract, the microbrewery placed its orders on the first of each month for either four or five barrels of barley. At the beginning of the third year of the contract, an article about the microbrewery appeared in a national newspaper, causing its popularity to soar. The following month, the microbrewery placed an order for 20 barrels of barley. The farm could not meet the increased demand and refused to deliver the 20 barrels. The microbrewery sued the farm for breach of contract. Will the microbrewery be successful in its suit? (A) No, because its order for 20 barrels of bar- ley was unreasonably disproportionate to its previous orders over a two-year period. (B) No, because the quantity term in a contract for a sale of goods must be certain. (C) Yes, because its order for 20 barrels of barley reflected its actual requirement for that month. (D) Yes, because its change in demand was unforeseeable when the parties entered into the contract.

(A) The farm will prevail. Under U.C.C. section 2-306(1), quantities subject to requirements contracts may not be unreasonably disproportionate to any stated estimate or, in the absence of any stated estimate, to any normal or otherwise comparable prior requirements. Here, the January order was four to five times larger than the previous orders over a two-year period and, therefore, the size of this order is unreasonably disproportionate to comparable prior requirements. Thus, (A) is correct. (B) is incorrect because a requirements contract is a special case. Even though no specific quantity is mentioned in an offer for a requirements contract, it is sufficiently definite because the quantity term is capable of being made certain by reference to objective, extrinsic facts (i.e., the buyer's actual requirements). (C) is incorrect because, although the buyer's requirements are measured by actual requirements as may occur in good faith, under U.C.C. section 2-306(1), they may not be unreasonably disproportionate to any stated estimate or to any normal or otherwise comparable prior requirements. (D) is incorrect because the rule regarding unreasonably dispro- portionate orders must be followed even when the change in demand was unforeseeable when the parties entered into the contract.

A purchasing agent of a steel company invited bids from several coal companies for 10 lots of metallurgical coal. The coal company with the winning bid offered $750,000, which was substantially under the market price for metal- lurgical coal. The agent immediately made a written agreement with the coal company to carry out the purchase, but several weeks before the coal was to be delivered, the coal company notified the agent that it had made an arithmet- ical error in pricing the coal and that the price should have been $1 million. The coal company requested a price increase to $825,000, which the agent approved in writing. If the agent had no reason to believe that the coal company had made an error in its initial calculations, is the modification by which the steel company agreed to pay $825,000 enforce- able? (A) Yes, because the request for the modifica- tion was made in good faith. (B) Yes, only if the coal company can estab- lish that enforcement of the contract at the $750,000 price would be unconscionable. (C) No, because the coal company was under a preexisting duty to sell at $750,000. (D) No, unless the arithmetical error is tanta- mount to changed circumstances.

(A) The modification is enforceable because it was made in good faith. Article 2 of the U.C.C. applies because coal is a movable good. Under section 2-209 of the U.C.C., consideration is not required for an enforceable modification; however, any modification will be subject to the general Code requirement of good faith and fair dealing, which requires honesty in fact and conformity with reasonable commercial standards. It appears that the coal company was being honest in informing the agent of the clerical error and that both parties made the modification freely. There is no indication that the modified price is commercially unreasonable, despite the required assumption that the disparity was not sufficient to indicate any mistake to the agent. (B) is incorrect because it is not true that the law will only support modifications of otherwise unconscionable contracts. (C) is incorrect because U.C.C. Article 2 abolishes the preexisting duty rule and the requirement of additional consideration to support a modification. (D) is incorrect because Article 2 does not require changed circumstances in order to support a valid modification. All that is required is good faith.

A homeowner mailed a letter to a prospective buyer on January 15 offering to sell her house for $25,000. The letter was delivered to the buyer on January 17. The buyer mailed a letter to the homeowner on January 19 stating that she accepted the offer. The buyer's letter of January 19 operates as an acceptance even under which of the following circumstances? (A) (B) (C) (D) The homeowner telephones the buyer on January 18 telling Buyer that she is revok- ing her offer. The buyer's letter is lost by the post office. The buyer misaddressed the envelope and, as a result, it was not delivered to the homeowner until February 1, after the homeowner had sold the property to a third party. The buyer knew that the homeowner had sold the property to a third party on January 18.

(B) Even if the post office loses the acceptance, a contract is formed. Under the mailbox rule, the acceptance becomes effective when the letter is put out of the possession of the offeree, i.e., when it is properly posted. This is true if the offeror at least implicitly authorized the offeree to use the mails in accepting and the mails are a reasonable means of responding to the offer. Here, the homeowner's mailing of her offer authorized the buyer to use the same method in accepting. (A) is incorrect. If the homeowner telephones the buyer on January 18, revoking the offer, the revoca- tion is good and the attempted acceptance on January 19 is too late. The mailbox rule applies here, and stands for the principle that the acceptance takes effect as soon as it is put out of the possession of the offeree. Thus, the buyer's acceptance would only become effective when she mails it on January 19, which is too late under the additional facts presented in this choice. Unless the offer is irrevocable or a firm offer, the offeror retains the right to revoke the offer at any time prior to acceptance by the offeree. Such revocation is effective upon receipt by the offeree, which is January 18 here. (C) is incorrect. Under the mailbox rule, the acceptance becomes effective when the letter is properly posted. However, where, as here, the envelope is misaddressed, then the rule that the acceptance becomes effective at the time of posting no longer applies. In such a case, the acceptance becomes effective only when received, which is too late in this case. (D) is incorrect because an offeree cannot accept the offer when the offeree knows or has reliable information that the offeror has done some act, usually selling the property, which is inconsistent with an outstanding offer. Revocation of an offer may occur prior to the time of acceptance when the offeree acquires information which would indicate to a reasonable person that the offeror no longer wishes to keep the offer open. Here, the buyer's knowledge that the homeowner sold her house to a third party before the buyer accepted the offer means that the offer was revoked, and the buyer's power of acceptance was terminated.

A parks and recreation association placed an order for 100 12-inch softballs with a softball manufacturer. The order stated that the associa- tion would pay the current market price for the softballs and requested prompt shipment. The manufacturer promptly shipped 100 16-inch softballs to the association because the manufac- turer's shipping director negligently misread the size listed on the shipping instructions. Which of the following best describes the association's rights and duties upon arrival of the 16-inch softballs? (A) A contract was formed by the manufac- turer's prompt shipment of the 16-inch softballs and the association must accept the softballs and offset its damages against the purchase price. (B) A contract was formed by the manufac- turer's prompt shipment of the 16-inch softballs, but the association is not required to accept the softballs and may sue the manufacturer for its damages. (C) The manufacturer's prompt shipment was a counteroffer and the association can accept the softballs or reship them to the manufac- turer because it has a duty to mitigate damages. (D) The manufacturer's prompt shipment was a counteroffer, and the association can refuse to accept the softballs, but has no cause of action for damages.

(B) The association may reject the softballs and sue for damages. Softballs obviously are movable goods; therefore, Article 2 of the U.C.C. applies. According to section 2-206(1)(b), an order for "prompt shipment" may be accepted by shipment of either conforming or nonconforming goods, and a contract is created upon such shipment. This alters the traditional rule that a shipment of nonconforming goods was a counteroffer which the buyer accepted upon taking delivery. The association may reject the nonconforming shipment [U.C.C. §2-601] and sue for damages. It may also, if it wishes, accept the shipment [U.C.C. §2-601], notify the seller of the breach [U.C.C. §2-607(3)], pay the contract price [U.C.C. §2-607(1)], and seek damages even after accepting the nonconforming goods [U.C.C. §2-7l4]. (A) is incorrect because, while a contract was indeed formed by shipment under U.C.C. section 2-206(1)(b), the association is not required to accept shipment, but may reject it. [U.C.C. §2-601] (C) is incorrect because the shipment constitutes an acceptance, not a counteroffer, as described above. Additionally, even where a buyer right- fully rejects a nonconforming tender, the buyer is under no obligation to reship prior to receipt of reasonable instructions from the seller. [U.C.C. §2-603] (D) is incorrect because, as discussed above, the shipment constitutes an acceptance, not a counteroffer.

A dog owner owned a mutt of little monetary value. One day, the dog disappeared, and the owner placed the following advertisement in the newspaper: "I will pay $500 to anyone who returns my dog." A man found the dog the next day. Knowing to whom the dog belonged, the man took the dog home, intending to return it that night. Before he returned the dog, the man read the dog owner's advertisement in the newspaper. Assuming that the dog owner's advertisement was an offer, what are the rights of the man who found the dog? (A) He can recover $500, as he has accepted the offer. (B) He can recover $500 if he returns the dog to the dog owner. (C) He cannot recover the $500, even if he returns the dog, because he did not have knowledge of the offer when he found the dog. (D) He cannot recover the $500 because the dog owner's promise was not represented by a signed memorandum.

(B) The man can recover $500 if he returns the dog. The man has not accepted the offer until he performs the requested act, i.e., returning the dog. Once he returns the dog, he has not only accepted the offer but fully completed his performance. At that point, the dog owner must perform his obligation under the contract to pay the man who found the dog $500. (A) is incor- rect because it fails to make clear that the man who found the dog must return the dog before he has a right to the money. Finding the dog alone does not entitle the man who found the dog to the reward money. The dog owner has made a unilateral offer, i.e., one that requires acceptance by the doing of an act. The act required for acceptance is the return of the dog. Until the man who found the dog actually returns the dog, he is not entitled to the reward money. (C) is incorrect because the finder's knowledge at the time of finding the dog is not material; only his knowledge at the time of performing the act required for acceptance (i.e., returning the dog) is relevant. Perfor- mance of an act requested by an offer made before the offeree has knowledge of the offer does not form a contract; however, where, as here, the offeree has only partially completed performance (finding the dog), completion of the requested performance (returning the dog) with knowledge acts as acceptance. If the man who found the dog had also returned the dog without knowing of the dog owner's offer of a reward for his return, he would have no contractual claim for the reward money. (D) is incorrect because this contract does not fall within the Statute of Frauds. The Statute of Frauds provides that certain contracts are enforceable only if they are evidenced by a written memorandum, signed by the party to be charged. The Statute of Frauds would cover a contract for the sale of goods where the price is $500 or more, but this question does not involve a sale of goods, because the man who found the dog does not have title to the dog. Neither does this transaction come under any other provision of the Statute of Frauds. Therefore, no written signed memorandum is necessary to the dog finder's recovery.

A shoe retailer ordered by telephone 12 pairs of dress shoes from a shoe manufacturer at its list price of $500 per dozen; delivery in 20 days. The manufacturer orally accepted the offer, and immediately faxed to the retailer this signed memo: "Confirming our agreement today for your purchase of a dozen pairs of dress shoes for $500, the shipment will be delivered in 20 days." Although the retailer received and read the manufacturer's message, he rejected the conforming shipment when it timely arrived. On learning of the rejection, does the manufacturer have a cause of action against the retailer for breach of contract? (A) Yes, because the shoes were identified to the contract and tendered to the retailer. (B) Yes, because the manufacturer's faxed memo to the retailer was sufficient to make the agreement enforceable. (C) No, because the agreed price was $500 and the retailer never signed a writing evidencing a contract with the manufac- turer. (D) No, because the retailer neither paid for nor accepted any of the goods tendered.

(B) The manufacturer has a cause of action for breach of contract. Because the contract was for goods priced at $500, the Statute of Frauds must be satisfied under U.C.C. section 2-201. Because both the retailer and the manufacturer are merchants, a memorandum of the terms of the sale sent by the manufacturer to the retailer satisfies the Statute of Frauds unless the retailer objects within 10 days, which she did not do. (A) is incorrect because identifying the goods to the contract and tendering the goods does not satisfy the Statute of Frauds. (C) is incorrect because, while the price makes the Statute of Frauds applicable, it is satisfied by the memorandum written by the manufacturer to the retailer, which was never rejected. (D) is incorrect because neither payment for nor acceptance of goods is necessary to make a contract enforceable, which it is here because the Statute of Frauds was satisfied by the memorandum sent by the manufacturer to the retailer.

A shopkeeper loaned a long-time employee $1,500 from his personal bank account because a family illness was causing the employee unexpected financial difficulties. Because the employee had proved himself to be trustworthy, there was no writing evidencing the loan and no payback date established; it was understood that the employee would repay the loan when he was able to do so. Sometime later, the shopkeeper's nephew asked him if he could help fund a business that he was starting up. Because most of the shopkeeper's assets were currently tied up, he asked his employee if he would be in a position to repay the $1,500 loan. The employee promised to repay the loan on the following Monday, so the shopkeeper told the employee to pay the $1,500 directly to his nephew. Immedi- ately thereafter, the shopkeeper informed the nephew to expect $1,500 from the employee on the following Monday. When Monday came, the employee decided he would rather tender the money to the shopkeeper than to someone he did not know, and the shopkeeper accepted the money. If the nephew never receives any money from the shopkeeper, will he succeed in an action against the employee for the $1,500? (A) Yes, because the shopkeeper effectively as- signed his right to collect the $1,500 to the nephew. (B) No, because the shopkeeper's acceptance of the $1,500 from the employee revoked the shopkeeper's gift to the nephew. (C) No, because the assignment was unsup- ported by consideration and therefore never effective. (D) No, because the employee's tender of the $1,500 to the shopkeeper and the shopkeep- er's acceptance of it constituted a novation.

(B) The shopkeeper validly assigned his right to receive the money to his nephew. However, this assignment was revocable, and it was revoked when the shopkeeper accepted the money from the employee. A creditor's right to receive money due from a debtor is a right that can be assigned, regardless of whether the debt is evidenced by a writing. By telling the employee to pay the money to the nephew, the shopkeeper manifested an intent to transfer his rights completely and immediately to the nephew. Neither a writing nor consideration was required for this assignment to be valid. However, these factors do not affect revocability. This assignment was not given for value. Such a gratuitous assignment is generally revocable. An exception to this rule arises when the assignor is estopped from revoking because he should reasonably foresee that the assignee will change his position in reliance on the assignment and such detrimental reliance occurs. Here, there is no indication that the nephew in fact changed his position detrimentally in reliance on the assignment. Consequently, the general rule of revocability of a gratuitous assignment applies. One way in which a gratuitous revocable assignment may be terminated is by the assignor taking performance directly from the obligor. By accepting the money from the employee, the shopkeeper (the assignor) took direct performance from the obligor, thereby revoking the assign- ment. As a result, the nephew has no right to the money. (A) is incorrect because it fails to account for the fact that, although the shopkeeper effectively assigned his right, he later revoked this assignment. (C) is incorrect because, as discussed in (B) above, a gratuitous assignment (i.e., an assignment not supported by consideration) is effective, although revocable except under certain circumstances not applicable here. (D) is incorrect because the facts do not indicate that there has been a novation. There is a novation when a new contract substitutes a new party to receive benefits and assume duties that had originally belonged to one of the original parties under the terms of the old contract. Here, the original agreement was between the employee and the shopkeeper. The employee's payment of the money to the shopkeeper and the shopkeeper's accep- tance thereof did not substitute any new parties or extinguish contractual duties as between the original contracting parties. Thus, there was no novation.

A golf pro entered into an employment contract with a country club to be its golf pro. The agreement specified that the golf pro would run the pro shop and provide private instruction to members from April through September of each year for the next five years, at a monthly salary of $5,000, plus instructional fees. During those months, the club's other instructor was playing on the professional tour and was unavail- able. On March 15, the club's manager received an e-mail from the golf pro, stating: "Made the final cut in Sarasota Winter Open. May not be able to get to club by April 1. Could be delayed until 5th if playoff necessary." The club's manager asked his attorney whether he should bring an immediate action against the golf pro for breach of contract. Which of the following is the most accurate advice for the manager? (A) Do not file suit; a repudiation must be in a signed writing to be of legal effect. (B) Do not file suit; the golf pro has not repudi- ated the contract. (C) File suit, but only if the club has changed its position to its detriment in reliance on the e-mail. (D) File suit, because the golf pro has repudi- ated the contract.

(B) This is a favorite MBE question. The best advice is not to file suit because the golf pro's e-mail does not constitute an anticipatory repudiation. Language may constitute an expression of doubt as to one's ability to perform under the contract without being an outright refusal. This will not be an anticipatory repudiation, but a prospective inability to perform. If there is an anticipatory repudiation, then the nonbreaching party can (i) sue for damages, (ii) contract with a third party, or (iii) do nothing. If the fact pattern language amounts to a prospective inability to perform, the innocent party may suspend performance until he receives adequate assurances that performance will be forthcoming. Here, the golf pro's e-mail does not constitute an anticipatory repudiation because he merely states that he "may not" get to the club by April 1. Thus, (B) is correct, and (D) is incorrect. (A) is an incorrect statement of law because a repudiation can be by any means: oral, written, or by actions. An e-mail message clearly stating an intent not to perform would be sufficient. (C) is also incorrect because the golf pro's actions constitute a prospective inability to perform, which allows the club to suspend performance to the golf pro and demand assurances; it does not result in an immediate breach.

A dealer in oriental rugs acquired an antique rug measuring 24 feet by 36 feet. A banker inspected the rug and orally agreed to buy it for the asking price of $65,000, provided he was successful in purchasing the house he was trying to buy, because it had a living room large enough to accommodate the rug. The sale agreement was later reduced to writing, but the provision concerning the purchase of the house was not included in the written agreement. If the banker is unsuccessful in acquiring the house he wants because the owner decided not to sell, and the dealer sues the banker for the purchase price, what is the most likely result? (A) The dealer will prevail because the original oral agreement need not be in writing to be enforceable. (B) The dealer will prevail because of the parol evidence rule. (C) The banker will prevail because he was unable to acquire the house he wanted. (D) The banker will prevail because the dealer is not entitled to specific performance.

(C) The banker will prevail because he could not acquire the house. In general, the parol evidence rule bars oral evidence contradicting a written agreement which was intended to be a final and exclusive embodiment of the parties' agreement. However, one exception to this general rule provides that parol evidence is admissible to show a condition precedent to the existence of a contract. Here, the contract between the banker and the dealer for the sale and purchase of the rug was only to be effective if the banker acquired the house he wanted. This condition precedent may be shown by the banker despite the fact that it was not reduced to writing. (A) is incorrect. This contract must be in writing because it is for the sale of goods of more than $500 in value. However, there is a written agreement here and the question is whether the written agreement precludes proof of the oral agreement. Under the parol evidence rule, a written agreement can prevent proof of an oral agreement even if the agreement does not have to be in writing. (B) is incorrect because a condition precedent to a written agreement's enforceability may be shown by parol evidence. (D) is incorrect because specific performance is not the issue here; whether the contract is enforceable at all is the issue. If this contract is enforceable, specific performance may be available because the rug might be considered unique.

An accountant offered to sell his motorcycle to a coworker for $10,000. The coworker told the accountant that he would think about it and get back to him. The accountant replied, "Okay, I will definitely hold onto the motorcycle until I hear from you." The following weekend, the accountant's brother-in-law offered him $11,000 for the motorcycle, and he sold it to the brother- in-law on the spot. On Monday, the coworker heard from a reliable source that the accountant had sold the motorcycle to his brother-in-law. The coworker immediately went to the accoun- tant's cubicle and told him, "I accept your offer." The accountant informed the coworker that it was already sold. If the coworker sues the accountant for breach of contract, will he win? (A) Yes, because the accountant did not ex- pressly tell the coworker that he had sold the motorcycle before the coworker ac- cepted his offer. (B) Yes, because the accountant made a firm offer to the coworker that was irrevocable, and the coworker accepted the offer. (C) No, because the offer to the coworker was terminated before he accepted it, when the accountant sold the motorcycle to his brother-in-law and the coworker learned of the sale. (D) No, because the accountant had the right to sell the motorcycle to a buyer who was willing to pay a higher price.

(C) The coworker will not prevail in his suit against the accountant, because the offer was effec- tively terminated by the indirect revocation of the offer that occurred when the accountant sold the motorcycle to his brother-in-law and the coworker learned of the sale from a reliable source. The revocation of an offer terminates the offeree's power of acceptance if it is communicated to him before he accepts. An offer may be effectively terminated if the offeree indirectly receives (i) correct information, (ii) from a reliable source, (iii) of acts of the offeror that would indicate to a reasonable person that the offeror no longer wishes to make the offer. This is what happened when the coworker heard from a reliable source that the accountant had sold the motorcycle. (A) is incorrect because, as explained above, the revocation of an offer can be indirect; it need not be express. (B) is incorrect because, notwithstanding his promise to hold onto the motorcycle until he heard from the coworker, the accountant's offer was not an irrevocable "firm offer" under the U.C.C. because the accountant was not a merchant and the offer was not in a signed writing. Nor did the coworker detrimentally rely on the accountant's promise or pay consideration to keep the offer open. (D) is incorrect because, had the offer been irrevocable—as a merchant's firm offer, because of consideration paid to keep the offer open, or because of detrimental reliance—the accountant would not have had a right to sell the motorcycle to another, even one who was willing to pay a higher price for it.

A debtor owed $50,000 to a creditor. One week before the statute of limitations was to expire, the debtor's mother sent the creditor a letter stating that she was sure that her son was about to pay the debt, and that if her son did not pay, she would. Relying on the mother's letter, the creditor allowed the statute of limitations to expire without bringing an action to recover the money. The debtor refused to pay the creditor. If the creditor brings suit against the mother for the $50,000, will he prevail? (A) No, because the statute of limitations has run against the son's debt. (B) No, because the mother received no bargained-for exchange to support her promise. (C) Yes, because the creditor reasonably and foreseeably relied on the mother's promise. (D) Yes, because of the main purpose exception to the statute of limitations.

(C) The creditor will prevail. The mother made a gratuitous promise to the creditor; she received no consideration for it (she did not ask for anything in return for her promise). The creditor's reliance in allowing the statute of limitations to lapse was not exchanged for the promise. However, even without consideration, under section 90 of the Restatement (Second) of Contracts, the creditor could enforce the mother's promise because he reasonably and foreseeably relied on the promise in declining to sue the debtor within the limitations period. (A) is incorrect because, although the statute of limitations has run on the debtor's debt, the creditor is not suing the debtor on his promise, but rather the mother on hers. (B) is incorrect because, while it is true that the mother received no consideration for her promise, the promise is enforceable under promissory estoppel principles. (D) is incorrect because there is no "main purpose" exception to the statute of limita- tions. The main purpose rule is an exception to the Statute of Frauds; however, that is not an issue here because the mother's promise was in writing.

A high-volume pleasure-boat retailer entered into a written contract to sell a customer a power boat for $120,000. The retailer could obtain from the manufacturer, for $90,500, as many of these boats as it could sell. As the contract provided, the customer paid the retailer $40,000 in advance and promised to pay the full balance on delivery of the boat. The contract contained no provision for liquidated damages. Prior to the agreed delivery date, the customer notified the retailer that he would be financially unable to conclude the purchase; the retailer thereafter resold the boat that the customer had ordered to a third person for $120,000 cash. If the customer sues the retailer for restitution of the $40,000 advance payment, which of the following should the court decide? (A) The customer's claim should be denied, be- cause, as the party in default, he is deemed to have lost any right to restitution of a benefit conferred on the retailer. (B) The customer's claim should be denied, because, but for his repudiation, the retailer would have made a profit on two boat sales instead of one. (C) The customer's claim should be upheld in the amount of $40,000 minus the amount of the retailer's lost profit under its contract with the customer. (D) The customer's claims should be upheld in the amount of $39,500 ($40,000 minus $500 as statutory damages under the U.C.C.).

(C) The customer should recover $40,000 minus the retailer's lost profit. The correct measure of damages is the lost profits of the retailer. That amount should be deducted from the deposit and the balance returned. The U.C.C. authorizes four different measures of damages when a buyer defaults after putting down a deposit. First, the Code allows the deposit to serve as liquidated damages if the contract so provides, within certain limits. However, this rule is inapplicable in this case because a liquidated damages clause was not included in the contract. Second, the Code provides in section 2-718(2) that when a deposit is given and there is no liquidated damages provision, the seller may keep 20% of the contract price or $500, whichever is less. In this case, the $500 limitation would control. However, if actual damages are more, as they are in this case, the seller may collect actual damages rather than the statutory amount. In section 2-708, the Code provides two measures of actual damages when the buyer defaults. The first is the tradi- tional expectancy measure of damages, resale price minus contract price. In a case such as this, there are no damages under this measure because the boat was resold for the contract price. The second alternative under section 2-708 applies here. When the seller is a dealer and the traditional measure of damages is inadequate to put him in as good a position as he would be in if the sale went through, then the measure of damages is lost profit. Here, since the dealer could have ordered as many boats as there were buyers, he actually lost his profit on this sale. (A) is incorrect because even a defaulting party has the right to recover a portion of the deposit back in restitu- tion if the amount of the deposit exceeds the contract damages to which the non-defaulting party is entitled. (B) is incorrect. While it is true that the retailer would have made profits on two boat sales instead of one if the customer had not defaulted, it did make the profit on the second sale, and cannot collect this from the customer. The lost profit on the first sale amounts to $29,500, less than the $40,000 advance. Therefore, the customer is entitled to $10,500, the difference between the amount of advance and the profit actually lost. (D) is incorrect because, as discussed above, actual damages allowed under U.C.C. section 2-708 exceed the $500 allowed under U.C.C. section 2-718(2), so the higher amount of damages would be awarded.

The father of a high school senior who had poor grades his junior year offered to pay his son $1,000 if he quit playing video games for the rest of the school year. The son agreed to quit that very day. However, shortly thereafter the son began to have doubts as to whether his father would really pay him the $1,000 if he kept his end of the bargain. The son told his mother about the conversation and she told him, "Go ahead and quit—if your father won't pay, I'll see that you get the money he promised you from my own account." The son quit playing video games for the rest of the school year, and he made the honor roll that year. Unfortunately, his parents did not live to see him complete this feat—they were killed in a skydiving accident two weeks before the end of the school year and one month before his 18th birthday. At the appropriate time, the son submitted a claim for $1,000 to the executor of his father's estate and was refused payment because the estate was insolvent after paying numerous secured creditors. The son then filed a claim against his mother's estate for the $1,000 based on the promise his mother made to pay that amount if his father refused to pay it. What is the best argument for the probate court's rejecting this claim against the mother's estate? (A) The contract between the mother and the son was illusory. (B) The son has not been damaged by any breach because the only effect—that he quit playing video games—was salutary. (C) The contract between the mother and the son was oral. (D) The son did not turn 18 years of age until two weeks after he completed performance and, thus, the contract was void because it lacked mutuality of obligation.

(C) The estate's best defense is that the contract was oral. Generally, contracts need not be in writing to be enforceable; however, under the Statute of Frauds, certain contracts must be evidenced by a writing signed by the party to be charged to be enforceable. Contracts in which one party promises to pay the debt of another, such as this contract, must be in writing. The mother promised to pay her son the amount owed to him by his father if the father did not pay him. Thus, there was a suretyship promise that had to be in writing under the Statute of Frauds, and this one was not. Therefore, (C) is correct. (A) is incorrect because the promise was not illusory. A promise is illusory when there is not consideration on both sides of the contract. Here, the son will receive $1,000 if he performs (i.e., refrains from playing video games for a specified time period), and the mother will receive the son's detriment of not doing something that he has a right to do, which is valid consideration (the benefit to the promisor need not have economic value). The son's performance is valid consideration even though he has already promised his father that he would give up video games (i.e., it is not a preexisting duty), because the son was not bound by his promise to his father. The father's offer was for a unilateral contract (i.e., one seeking perfor- mance rather than a promise to perform), and so could be accepted only by performance. The son had not yet performed when his mother made her promise to pay him if his father did not, so the son had not yet accepted his father's offer and was not bound by his promise to quit playing video games. Therefore, he was not under a preexisting duty, and his mother's promise served as additional consideration for the son's performance. Note also that a surety such as the mother will be bound by her promise to pay another's debt as long as she makes her promise before the creditor (the son) performs or promises to perform; the surety need not receive any separate consideration. (B) is incorrect because the son's giving up what he had a legal right to do—even if it is detrimental—is sufficient consideration to support a contract, so the mother is bound to pay even though the contract was beneficial to the son. (D) is incorrect because a contract between an infant (i.e., a person under the age of 18) and an adult is binding on the adult, even though it is voidable by the infant. Courts abide by the legal fiction of mutuality of consideration in cases involving infant-adult contracts for reasons of public policy, so such contracts may be made and acted on.

A weather vane collector placed the following ad in a newspaper: "I will pay $300 for informa- tion as to where I can purchase a Connecticut copper weather vane with Victorian Serpentine Motif in good condition." The ad, which included contact information, was placed on Friday, December 5, and was to run a full week, starting on Sunday the 7th. A friend of the collector's knew of his quest for the weather vane, but she did not see the newspaper ad. On Sunday morning, she saw at a swap meet the exact type of weather vane the collector sought and called him to tell him about it. The collector hurried down to the swap meet and got his weather vane at a good price. On Monday, December 8, the friend saw the collector's ad in an open newspaper. She called the collector and asked him to pay her $300 for the tip. After he declined, saying the offer was revoked when he purchased the weather vane, she sued the collector for her reward. Will she prevail? (A) Yes, because the collector bought the vane based on her tip. (B) Yes, because the collector's revocation was ineffective. (C) No, because the friend did not know of the offer when she accepted. (D) No, because the collector's ad was not an offer.

(C) The friend cannot recover because she did not know of the offer. Although the collector's place- ment of the ad was an offer, his friend did not accept by giving the requested information, because she did not know about the ad when she gave the collector the lead on the weather vane. The general rule on public offers is this: an offer of reward is an offer to enter into a unilateral contract, and if made to the public generally, it may be accepted by anyone to whom it becomes known. One who performs the requested act has done all that is necessary for acceptance, but if he does not intend that his acts constitute an acceptance, no contract results. Thus, where he has no knowledge of the offer, no act of his can be deemed an acceptance. [See Restatement of Contracts §§23 comment (c); 51 comment (a)] (A) and (B) are incorrect because they reach the wrong result; there is no contract with the friend for the reasons above. The fact that the revoca- tion may have been ineffective begs the issue, because even though the offer was still in existence until effectively revoked, there can be no contract without the proper form of acceptance as noted above. (D) states the rationale incorrectly; the friend loses because she did not accept, not because of a defect in the offer. This ad was specific enough to constitute an offer.

On September 1, an art collector offered to sell one of his expensive paintings to a buyer. The buyer, who was a friend of the collector, wanted a few days to make up her mind, so the collector and the buyer decided that the collector would keep his offer to her open until September 8 in exchange for a payment of $5. Later that week an art investor tendered to the collector double what he was asking for the painting. On the morning of September 8, the buyer telephoned the collector to tell him that she wanted the painting but his phone was out of order, so she wrote out a check for the agreed- on amount and dropped it into a mailbox before leaving town. On September 9 the collector, not having heard from the buyer, sold the painting to the investor. Who is entitled to the painting? (A) The buyer, because she had a valid option contract with the collector and effectively accepted the collector's offer to sell the painting. (B) The investor, because $5 is inadequate consideration for an option contract involving an expensive purchase such as this one. (C) The investor, because the collector sold the painting to the investor on September 9 before receiving the buyer's check. (D) The collector, because a sale of goods priced at $500 or more must be in writing to be enforceable.

(C) The investor owns the painting. While it is true that the buyer had a valid option contract with the collector, she did not effectively accept the collector's offer to sell the painting when she dropped the check into the mailbox on September 8. The majority view is that acceptance of an option is effective only when received by the offeror, so the usual "mailbox rule" does not apply to make the acceptance effective on dispatch. This means that the buyer did not effectively accept the option within the stated period, i.e., by September 8. When, as here, the time the offer will remain open is specified in the option, if it is not accepted within that time, the offer terminates due to lapse of time. (A) is incorrect because, as discussed above, the buyer does not own the painting because acceptance of an option is effective only when received by the offeror, and here the collector would not have received the buyer's acceptance until it was too late. (B) is incorrect because the consideration exchanged by the offeree (the buyer) for the offeror's (the collector's) promise not to revoke need not be "adequate"—generally, consideration of any value is sufficient to support an option. (D) is incorrect because the collector sold the painting to the investor. While this choice accurately sets forth the Statute of Frauds rule for the sale of goods, the outcome of the dispute here does not turn on this issue. Perhaps if the collector had changed his mind about selling the painting to the investor, he could assert that the Statute of Frauds required the agree- ment to be in writing to be enforceable. However, the facts do not present that situation, nor do they indicate whether the sale was oral or in writing.

A landscaper entered into a written contract with a developer to landscape a 30-house subdi- vision at a price of $4,000 for each house. The contract provided for payment of the $120,000 only on completion of the landscaping for all the houses. After completing 20 houses, the landscaper demanded payment of $80,000. The developer refused. The landscaper then walked off the job without doing any landscaping on the other 10 houses. What may the landscaper recover from the developer? (A) Nothing, because payment was expressly conditioned on completing the landscaping of all the houses. (B) The landscaper's expenditures plus antici- pated profit in landscaping the first 20 houses up to a maximum recovery of $80,000. (C) The reasonable value of the landscaper's services in landscaping the 20 houses, less the developer's damages for the failure to landscape the other 10 houses. (D) The total amount that the 20 houses have increased in value from the landscaping.

(C) The landscaper may recover the reasonable value of his services less the developer's damages. Unless there is an expressed condition that would reasonably be understood to provide for forfeiture of payment unless there is full performance, a breaching party may seek restitution damages for the fair value of any benefit conferred, the reasonable value of services provided. As a nonbreaching party, the developer has the choice of remedies. The developer may enforce the benefit of this bargain and elect to pay the landscaper the contract price ($120,000) minus the cost to have the last 10 houses landscaped. Alternatively, the developer may rely on the material breach as a reason to set aside the bargain, in which case the developer need only pay the landscaper the net benefit received, i.e., the reasonable value of the landscaper's services for the first 20 houses, less any damages incurred due to the landscaper's failure to perform. Note that the landscaper alternatively could make an argument that the contract is divisible, because a contract provision that payment is to be made only on completion of all units is often construed by the courts as merely stating a time for payment rather than a condition making the contract indivisible. However, since divisibility is not one of the answer choices, the restitution remedy in (C) is the best option. (A) is incorrect because paying the landscaper nothing would result in unjust enrichment of the developer. While the terms of the contract are no longer enforceable, the landscaper is entitled to payment based on substantial performance. The developer is entitled to an allowance for hiring someone else to complete the landscaping. Therefore, the measure of recovery is the promised price minus any amount it will cost the developer to have the remaining houses landscaped as the landscaper promised to do. (B) is incorrect. This answer has elements of the correct answer except that it does not contemplate the harm the developer may have suffered from the landscaper's refusal to complete the job. In establishing the value for restitution, the court must consider not only the benefit conferred (in this case from landscaping 20 houses), but also the injury resulting from the breach of the contract. (D) is incorrect because the increase of value, if any, is not relevant to value of the service provided. The landscaper is entitled to payment based on the reasonable value of the service provided in restitution for the fair value of any benefit conferred.

A mass marketer contracted with a political campaign to send out mass mailings to voters for $100,000. It subcontracted with a printer to print brochures for $20,000 over a period of several weeks. The printer would be paid on a weekly basis. After providing $15,000 of printing services the first few weeks, the printer unjustifiably refused to perform any additional work for the marketer. The marketer had paid the printer $10,000 to that point, and had to pay another printer $12,000 to print the balance of the brochures. The marketer sued the printer for breach of contract, and the printer counter- claimed for the reasonable value of the benefits conferred on the marketer and not paid. What will be the outcome of this litigation? (A) The printer should recover $5,000, the ben- efit conferred on the marketer for which the printer has not been paid. (B) The printer should recover $3,000, the benefit conferred on the marketer less the $2,000 in damages suffered by the marketer. (C) The marketer should recover $2,000, the excess it had to pay over the contract price to get the performance the printer had promised. (D) Neither party should recover anything, because the printer was in breach of contract and the marketer received $15,000 in printing services from the printer for $10,000.

(C) The marketer should recover $2,000. The primary objective of contract damages is to put the nonbreaching party in the same position that he would have been in had the contract been performed. The normal measure of damages is expectation damages. The marketer has a legally enforceable right to have the work under the printer contract performed for $20,000. Because the marketer paid the printer $10,000 and needed to spend $12,000 to have the printing completed by a third party, the marketer has spent $22,000. This establishes the marketer's right to seek $2,000 from the printer for its breach. (A) and (B) are incorrect because the purpose of contract damages is to put the nonbreaching party where he would have been had the promise been performed. For the printer to recover anything, he would have to prove that he is entitled to restitution. The breaching party will prevail in a restitution action only if the nonbreaching party seeks to keep the value of the benefit conferred without paying and, therefore, is unjustly enriched. The marketer had to pay $2,000 more than the price stated in the contract with the printer. The marketer was not unjustly enriched and, in fact, had damages of $2,000 from the breach. The printer's costs are not relevant. The printer is not entitled to recover anything. (D) is incorrect. To provide no remedy to either party on the theory stated in (D) is to make the marketer pay a total of $22,000 for the performance that was promised by the printer for $20,000. The primary objective of contract damages is to put the nonbreaching party in the same position that he would have been in had the contract been performed. Therefore, this theory fails.

An owner of a piece of waterfront property contracted in writing with a contractor to rebuild the owner's dock in accordance with plans and specifications prepared by the owner. The agreed contract price was $50,000, $25,000 of which was payable on May 1 when the job was to commence and the balance due upon completion of the work. On March 1, the contractor notified the property owner that the contractor would lose money on the job at that price, and would not proceed with the work unless the property owner agreed to increase the price to $80,000. The property owner did not respond to the contractor, instead making a written contract with a third party to repair the dock, commencing May 1, for $60,000, which was the fair market cost of the work to be done. On May 1, both the contractor and the third party showed up at the dock to begin work, the contractor telling the property owner that he had decided to take the loss and would repair the dock for $50,000 as originally agreed. The property owner dismissed the contractor and allowed the third party to begin work on the dock. In a contract action by the contractor against the property owner, which of the following would the court decide? (A) The contractor will win because the prop- erty owner did not tell him before May 1 about the contract with the third party. (B) The contractor will win because he attempted to perform the contract as origi- nally agreed. (C) The property owner will win because the contractor in legal effect committed a total breach of contract. (D) The property owner will win because the third party's contract price was $20,000 lower than the $80,000 demanded by the contractor on March 1.

(C) The property owner will win because the contractor breached. When the contractor notified the property owner on March 1 that he would not perform his obligations on a binding contract unless he was given more money, he committed an anticipatory breach. That breach gave the property owner the right both to terminate the contract and to engage a new contractor to complete the work. The property owner was not required to give notice to the contractor to exercise that right, which is why choice (A) is incorrect. (B) is incorrect because, when the contractor notified the property owner on March 1 that he would not perform his obligations on a binding contract unless he was given more money, he committed an anticipatory breach. That breach gave the property owner the right to terminate the contract and engage a new contractor to complete the work, without giving notice to the contractor. The contractor had no right to perform the contract once he had breached it; therefore, the fact that he showed up ready to perform the work on the day required by the contract is irrelevant. (D) is incorrect because the price at which the property owner was able to obtain a replacement contractor has no bearing on whether the property owner had the right to terminate the contractual rights of the original contractor. Indeed, after the contractor's breach, he could have chosen not to have the dock repaired at all, or if the lowest bid was from someone who would only do the work for more than $80,000, he could have accepted that bid and charged the contractor for the difference between the contract price and the cost of the cover.

A manufacturer and a buyer entered into a written contract for the manufacturer to produce and sell to the buyer 2,000 widgets at a price of $20 per widget. The contract expressly provided that the buyer shall have no liability under the contract unless 2,000 widgets are delivered to the buyer at his place of business no later than July 1. On July 1, 1,800 widgets meeting the buyer's specifications were tendered by the manufacturer. The remaining 200 widgets were tendered on July 5. The buyer refused to accept any of the widgets. In an action by the manufacturer against the buyer, which of the following would best support the manufacturer's case, assuming it can be proven? (A) The buyer had orally agreed, just prior to the time the written contract was executed, to accept and pay for partial deliveries of the widgets. (B) Widgets are a unique product produced only by the manufacturer and in a size and tolerance that varies with the needs of each purchaser. (C) Delivery of the 200 widgets on July 1 was delayed by a storm which disrupted the shipper's activities, and was not the manufacturer's fault. (D) A drop in the buyer's credit rating from good to fair had caused the manufacturer not to produce and tender the full 2,000 widgets on or before July 1.

(C) The storm delay would best support the manufacturer's case. The storm may have made delivery on time impossible, which may excuse performance under either the doctrine of impracticability of performance or the doctrine of impossibility of performance. These defenses can be used not only to excuse performance totally, but also to excuse the delay in performance. (A) is incor- rect because an oral agreement made prior to a written agreement on the same subject will not be admissible to alter the written agreement under the parol evidence rule. Therefore, the oral agreement does not modify the terms of the written contract and this contention will not help the manufacturer's case. (B) is incorrect. The fact that goods are specially made creates an exception to the Statute of Frauds, but it does not excuse the obligation to tender conforming goods in the correct quantity at or before the time specified for delivery. The perfect tender rule is incorporated into every contract for the sale of goods except where the contract specifies different terms or where there is an explicit installment contract. (D) is incorrect. If the buyer became insolvent, the manufacturer could, under the U.C.C., require that the buyer pay cash upon delivery or give assur- ances of payment. Here, however, the manufacturer has not requested that payments be made in cash or that the buyer give assurances. Moreover, the buyer only suffered a decrease in his credit rating; he is not insolvent. Therefore, the manufacturer would have had no basis for making either request. The manufacturer is not excused from performance by the buyer's decreased credit rating.

A catering company entered into a written contract with a dish supplier to purchase 5,000 plastic dishes at $.10 per dish. The contract called for the supplier to deliver the 5,000 dishes to the caterer on or before October 1. On October 1, the supplier delivered only 3,000 dishes to the caterer. The supplier informed the caterer that it was experiencing manufacturing delays and would deliver the other 2,000 dishes by October 31 at the latest. The caterer accepted delivery of the 3,000 dishes, but because it had a number of catering jobs lined up for early October, the caterer was forced to purchase 2,000 dishes from another supplier at a price of $.12 per dish. The supplier demanded that the caterer pay $300 for the 3,000 dishes delivered, but the caterer refused to pay anything. If the supplier sues the caterer for breach of contract, what will the supplier recover? (A) $300, the price under the contract for the 3,000 dishes that were delivered, with no deduction for the caterer's extra cost, because the caterer waived its right to cover when it accepted the supplier's tender with- out expressly reserving its rights. (B) The reasonable value of the 3,000 dishes that were delivered less $40, which is the extra cost incurred by the caterer to obtain the balance of the dishes. (C) $260, the price under the contract for the 3,000 dishes that were delivered less $40, which is the extra cost incurred by the caterer to obtain the balance of the dishes. (D) Nothing, because the supplier is in material breach of the contract until it tenders delivery of the last 2,000 dishes.

(C) The supplier will recover $260. Dishes are movable goods, and so Article 2 of the U.C.C. applies. Under Article 2, when goods fail in any respect to conform to the contract, the buyer may accept the goods [U.C.C. §2-601] and pay the contract price for the goods accepted [U.C.C. §2-607]. However, the buyer has a right to offset its damages. When a seller fails to deliver goods as promised, the buyer may "cover" under U.C.C. section 2-712 by making a reasonable purchase of substitute goods, and then may recover as damages the difference between the contract price and the "cover" price. Here, the difference between the contract price and the cover price is $40 (2,000 plates × $.02). (A) is incorrect because the U.C.C. permits a buyer to accept a noncon- forming or partial delivery without waiving the right to sue for damages; thus, there is no need for an express reservation of rights at the time the goods are accepted. (B) is incorrect because the correct measure of damages is not based on the reasonable value of the goods accepted, but rather on the contract price. [See U.C.C. §2-607(1)] (D) is incorrect because the U.C.C. requires that the buyer pay for any goods accepted. [U.C.C. §2-607] The common law rule, which discharges the buyer's obligation to pay if there has been a material breach, is displaced by U.C.C. section 2-607.

The proprietor of a men's shop contracted on May 1 with a wholesaler of shirts to buy 300 men's long-sleeve white cotton shirts for $4 per shirt, or $1,200. The parties each signed a purchase order calling for delivery of the shirts to the proprietor's place of business and payment of the purchase price on July 1. Two days later, the wholesaler discovered that he had made a mistake in his price quote. He told the propri- etor that, unless the proprietor paid him $8 per shirt, or a total price of $2,400, he would not be able to deliver the shirts. The proprietor refused to pay anything more than $1,200. When the wholesaler failed to deliver the shirts on July 1, the proprietor purchased 300 comparable shirts for $8 per shirt and brought suit against the wholesaler for damages. What is the likely result of this lawsuit? (A) The wholesaler will prevail because he made a unilateral mistake. (B) The wholesaler will prevail because of the doctrine of mutual mistake. (C) The wholesaler will prevail if the propri- etor had reason to know on May 1 that the wholesaler made a mistake in quoting the price of $4 per shirt. (D) Theproprietorwillprevail.

(C) The wholesaler will prevail if the proprietor had reason to know of the mistake. Where one party makes a unilateral mistake about a basic assumption on which the contract is based, and the other party knew or had reason to know of the mistake, the mistaken party will be allowed to rescind the contract. In this case, there is a substantial disparity between the contract price of $4 per shirt and the market value of the shirts at wholesale, which seems to be $8 per shirt. If the proprietor, because of the disparity in price, was or should have been aware that the wholesaler had made a mistake, the wholesaler will be able to rescind the contract and therefore will prevail. Hence, choice (D) is incorrect. (A) also is incorrect. This is a case where the wholesaler made a unilateral mistake in quoting the wrong price to the proprietor. Making a unilateral mistake alone, however, is not sufficient to allow the mistaken party to rescind the contract. A contract can be rescinded for unilateral mistake only when the other party knew of the mistake or when the mistake was so obvious that the other party should have known that the first party made a mistake. (B) is incor- rect because there is no mutual mistake. Mutual mistake occurs when both parties to a contract are mistaken about existing facts relating to a basic assumption on which the contract is made. Unilateral mistake occurs when there is a mistake by only one of the two parties as to a basic assumption upon which the contract is made. The facts of this question present a typical case of unilateral mistake, evidenced by the wholesaler's quoting the wrong price for the shirts. There is no mistake on the part of the proprietor, and the wholesaler will be able to rescind the contract only if it can be shown that the proprietor knew or should have known of the wholesaler's mistake.

After a difficult divorce, a mother wrote to her son and daughter the following: In consideration of your emotional support for me during that trying time and your love and affection for me, I promise to divide my estate between you in equal shares. You know you can count on your mother's word. The daughter thereafter continued her usual practice of calling her mother once a week and visiting her at Christmas and on her birthday until her mother died three years later. Shortly after the funeral, the daughter learned that the mother's will made the son the sole legatee. If the daughter sues the executor of the mother's estate for one-half of that estate, based on the mother's letter to her, will she win? (A) Yes, because she relied to her detriment on her mother's promise by visiting her mother. (B) Yes, because her emotional support of her mother during the divorce proceed- ings constituted valid consideration for her mother's promise. (C) Yes, because even though her mother's letter is a promise to make a gift in the future, the promise is in writing and intended by her to be enforceable and there- fore needs no consideration. (D) No, because the mother's promise was not supported by consideration.

(D) The daughter will lose because there is no consideration to support the promise. Promises to make gifts in the future are unenforceable even if they are in writing and are intended by the promisor to be enforceable. Hence, (C) is incorrect. A promise to make a gift does not involve a bargained- for exchange, and the requirement of consideration is not fulfilled. (A) is incorrect. This answer states the principle of promissory estoppel. Under the doctrine of promissory estoppel, where there is substantial detrimental reliance by a party on a promise of the promisor, the promise will be enforceable even absent consideration. It is unlikely that the daughter's continuation of her calling and visiting her mother would constitute substantial detrimental reliance. Absent consid- eration or a substitute, the mother's original promise would not be enforceable, and her daughter would be unsuccessful in her suit against the executor. (B) is incorrect because the emotional support given to the mother by her daughter was not bargained-for consideration. It was volun- tarily given before there was any promise to leave property by will, and therefore does not make the promise enforceable. In order to be part of the bargain, the element of consideration must be part of the bargained-for exchange. Thus, if the mother had said to her daughter, "If you will give me emotional support, I will leave you half my estate," the emotional support thereafter given by the daughter would have been bargained for by the mother in exchange for part of her estate. Because the daughter gave her support gratuitously before the promise, the mother's promise did not induce the legal detriment, and was therefore not supported by consideration.

A large producer of bread wrote to a distrib- utor of flour, asking, "How much will you charge to supply my needs for flour for the next year?" The distributor replied in writing that it could supply the producer with all the flour it would need next year at a specified price per pound. The producer wrote back, "Your offer to supply me with flour is hereby accepted, provided that you agree to a 10% discount if payment is made within 10 days from date of billing." What should the producer's reply concerning a 10% discount be characterized as? (A) An acceptance with a proposal for an ad- ditional term, provided both parties are merchants. (B) An acceptance with a proposal for an additional term, whether or not both parties are merchants. (C) A counteroffer, but not a rejection. (D) A rejection and counteroffer.

(D) The producer's reply likely would be a rejection and counteroffer. This question deals with the "battle of the forms" provision of the U.C.C., and is a recurring favorite MBE question. Under section 2-207 of the U.C.C., an acceptance containing additional or different terms is effective as between merchants unless the offeree expressly makes his acceptance conditional on assent by the offeror to the additional terms. In this fact pattern, because the producer made his purported acceptance conditional on the distributor's assent to the 10% discount, his communication would be a rejection and a counteroffer. Hence, (D) is correct and (A), (B), and (C) are incorrect. Note that if the producer's acceptance were not expressly conditional on assent to the discount, the discount would become part of the contract because they are merchants, unless (i) it materially altered the original terms of the offer (i.e., changed a party's risk or the remedies available); (ii) the offer had expressly limited acceptance to the terms of the offer; or (iii) the offeror objected to the terms within a reasonable time after notice of it.

On June 1, a homeowner sent a letter to a painter that stated, "I will pay you $8,000 for painting the exterior of my home, provided you commence painting by July 1 and finish by July 10." On June 5, the homeowner received the following letter from the painter, "I am available to paint your house, but I cannot do the work for less than $10,000." On June 10, the homeowner received the following letter from the painter, "I have changed my mind. I will do the work for $8,000. I will start on July 1 and finish painting before July 10 unless I hear from you differ- ently." The homeowner never responded to the painter's letter. On June 30, the homeowner left the country for a one-month vacation in Europe. Unbeknownst to the homeowner, the painter commenced painting the house on July 1. At this point, does the homeowner have a contractual obligation to pay the painter $8,000 on completion of the painting? (A) Yes, provided the painter completes the painting by July 10. (B) Yes, because the painter accepted the contract before the homeowner materi- ally changed his position in reliance on the painter's rejection. (C) No, because the different terms in the offer and the acceptance are knocked out, and the court will supply the missing price term. (D) No, because the homeowner left the country without ever accepting the painter's offer to do the work as originally requested.

(D) Thehomeownerisnotobligatedtopaythepainter.Thepainter'sJune5communicationwasa rejection of the offer for employment by the homeowner on June 1. The painter's June 10 communi- cation, therefore, created a new offer that would require some further manifestation of acceptance by the owner in order to form a contract. Note that the language "unless I hear from you differ- ently" has no effect on the outcome. Thus, (A) and (B) are wrong. (C) is wrong because it states a battle of the forms provision under the U.C.C., which does not apply because this is not a contract for the sale of goods. Under the common law mirror image rule, the acceptance must contain the same terms as the offer. Any variance results in a rejection and counteroffer. The painter may have a cause of action against the homeowner under an unjust enrichment theory. However, the question asks if the homeowner has a contractual obligation to pay, to which the answer is clearly no.

A gourmet food company entered into a long- term contract with an airline, under which the food company would supply the airline with 5 million gourmet dinners over a five-year period at a special rate of $2 per unit. The food company insisted as a term of the contract that the airline agree to purchase from a microwave supplier, and to install in each of its planes, a microwave oven specifically designed to heat frozen dinners, in part because the food company owned consid- erable stock in the microwave supplier. The contract between the food company and the airline had a clause that authorized "oral modifi- cations by the contracting parties." One month after the contract was signed but before any dinners were delivered, the airline informed the food company that it would have difficulty complying with the provision requiring purchase of the supplier's microwaves because the supplier's products had increased dramati- cally in price. Subsequent negotiations between the food company and the airline led to an oral agreement to increase the price per dinner to $2.08 per unit and eliminate the supplier's microwave requirement. If the supplier sues the airline for enforce- ment of the contract, what will be the most likely result? (A) Judgment for the supplier, because under the U.C.C. the oral modification is not valid and the supplier's rights vested when it brought suit. (B) Judgment for the supplier, because it was a third-party beneficiary to the original contract and it did not agree to the modifi- cation. (C) Judgment for the airline, because the contract was between the airline and the food company, and by its terms an oral modification was permissible. (D) Judgment for the airline, because the supplier's rights had not vested when the modification took place.

(D) This is a favorite MBE question. The airline will prevail because the supplier's rights had not vested at the time of the modification. The rights of the third-party beneficiary do not vest until: (i) it manifests assent in a manner invited or requested by the parties; (ii) it learns of the contract and detrimentally relies on it; or (iii) it brings a lawsuit to enforce its rights. Until a third party's rights have vested, a modification of the contract can take place without the consent of the third party. The proper analysis is that the supplier's rights have not vested. Even if the supplier was aware of the contract, and even if the supplier was pleased with it, the supplier had not assented to the contract in any manner, had not detrimentally relied, and had not brought suit on the contract before it was modified. Therefore, the supplier's rights had not vested, and the contracting parties were free to modify the contract. Moreover, notwithstanding the oral modification clause, while the oral modification will be enforceable between the food company and the airline only if an exception to the Statute of Frauds applies (e.g., the parties admit the modification or perform the contract as modified), it is doubtful that a court would allow the supplier to use the Statute to prevent the parties from admitting their modification. Thus, the modification is valid, and (A) is wrong. (B) is wrong because the supplier's rights had not vested. (C) is a true statement, but (D) is a better answer, because it states the specific reason that the supplier's rights had not vested.

A communication will not be considered to be definite and certain enough to be an offer if it is for the sale of goods and: A is missing a quantity term B is missing the price term C states the quantity to be purchased and sold as "all that the buyer requires" D states the quantity to be purchased and sold as "all that the seller produces"

A communication will not be considered to be definite and certain enough to be an offer if it is for the sale of goods and is missing a quantity term. The quantity term is the only term that is absolutely required to make a communication an offer when the sale of goods is involved. Most other terms can be implied or supplied later in the contract. A communication may be considered definite enough to be an offer for the sale of goods despite a missing price term. If the price term is not included, a reasonable price can be implied. The buyer's requirements and the seller's output are valid quantity terms sufficient to make a communication an offer for the sale of goods. Although these terms do not state a specific quantity, the quantity is capable of being made certain by reference to objective, extrinsic facts (i.e., the buyer's actual requirements and the seller's actual output).

What is a "constructive" condition? A A contractual provision providing that a party does not have a duty to perform unless some event occurs or fails to occur B A condition that is implied by a court even though it is not explicitly stated in the contract C A condition commonly found in construction contracts stating that the condition of complete performance may be excused if the party has rendered substantial performance D A contractual provision providing that the contract is not effective unless some event occurs or fails to occur

A constructive condition is a condition that is implied by a court even though it is not explicitly stated in the contract. Common examples of constructive conditions are the conditions of cooperation and notice. Constructive conditions are also known as implied conditions. In contrast, an express condition precedent is an explicit contractual provision providing that a party does not have a duty to perform unless some event occurs or fails to occur. When an entire contract is not effective unless some event occurs or fails to occur, the contract is subject to an express condition precedent, not a constructive condition. The concept of substantial performance was developed in construction cases to avoid the harsh results that could occur when complete performance is required. Under the doctrine of substantial performance, the condition of complete performance may be excused if the party has rendered substantial performance.

Which of the following statements regarding revocation and acceptance of contract offers is correct? A A revocation generally is effective when received, and an acceptance generally is effective when received. B A revocation generally is effective when received, and an acceptance generally is effective when dispatched. C A revocation generally is effective when dispatched, and an acceptance generally is effective when dispatched. D A revocation generally is effective when dispatched, and an acceptance generally is effective when received.

A revocation generally is effective when received and an acceptance generally is effective when dispatched (i.e., the mailbox rule). Under the mailbox rule, if the offeree dispatches an acceptance before he receives a revocation sent by the offeror, a contract is formed.

Unless properly disclaimed, this warranty is included in every contract for the sale of goods: A Warranty against infringement B Warranty of title C Implied warranty of merchantability D Implied warranty of fitness for a particular purpose

A warranty of title is included in every contract for the sale of goods. Any seller of goods warrants that the title transferred is good, that the transfer is rightful, and that there are no liens or encumbrances against the title of which the buyer is unaware at the time of contracting. The warranty against infringement, which warrants that goods are delivered free of any patent, trademark, copyright, or similar claims, arises automatically only in contracts by merchant sellers. The implied warranty of merchantability, which generally warrants that the goods are fit for the ordinary purpose for which such goods are used, also is implied only in contracts by merchant sellers. The implied warranty of fitness for a particular purpose arises only when: the seller has reason to know the particular purpose for which the goods are to be used and that the buyer is relying on the seller's skill and judgment to select suitable goods, and the buyer in fact so relies.

The owner of a house put the property up for sale. A surgeon entered into negotiations with the owner to purchase the house, and the parties agreed upon a sale price of $200,000. The owner told the surgeon that she would drop a contract in the mail and have her attorney draw up a deed. The owner signed a land sale contract, which included the property's address but did not contain a metes and bounds legal description. She mailed the contract to the surgeon that afternoon, although it was mailed too late for the last mail pickup of the day. The owner's attorney promptly drew up a deed and dropped it in the mail to his client, who did not sign it. The surgeon received the contract the next day. After she mailed the contract, the owner received an offer of $250,000 for her property from her next-door neighbor, who wanted to expand beyond his own property line. The owner called her attorney and told him to inform the surgeon that the deal was off. The attorney sent a letter to the surgeon, stating that his client had found another purchaser for the property, and that all matters regarding the surgeon's offer for the property were rescinded. The owner later received the signed contract from the surgeon. Can the surgeon compel the owner to convey the property to him for $200,000? A Yes, because the owner signed the land sale contract. B No, because the land sale contract does not contain the complete legal description of the property. C No, because the deed was not signed by the party to be charged. D No, because contracts involving land are governed by the Statute of Frauds.

A. The surgeon is entitled to specific performance because the owner signed the land sale contract. A contract was formed here when the parties orally agreed to the sale of the property. However, the contract was unenforceable at that time because, under the Statute of Frauds, a contract for the sale of land is unenforceable unless a memorandum containing the contract's essential terms is signed by the party to be charged. Here, the party to be charged is the owner, and she signed the land sale contract, a writing sufficient to satisfy the Statute of Frauds (a memorandum for the sale of land is sufficient if it contains the price, a description of the property - which need not be a "legal" description - and a designation of the parties). Thus, the contract was enforceable. Specific performance is allowed when the legal remedy (damages) would be inadequate (usually with contracts to purchase land). Therefore, the surgeon is entitled to specific performance (assuming the property has not already been sold to a bona fide purchaser); under the facts the neighbor had made an offer but nothing indicates that the owner accepted the offer yet. (B) is incorrect because to satisfy the Statute of Frauds, a description need not be a complete legal description, but need merely be sufficient to reasonably identify the subject of the contract. It is sufficient that the property was identified by its address. (C) is incorrect because it does not matter whether the deed was signed by the owner, because the land sale contract was sufficient under the Statute, and the owner signed it. (D) is incorrect because while it is true that contracts involving the sale of land are governed by the Statute of Frauds, the Statute was satisfied here by the written sale contract.

Which of the following best states the result when, in a contract between merchants for the sale of goods, the acceptance does not match the terms of the offer? A Additional terms are never included in the contract, but different terms will be included unless the offer expressly limits acceptance to the terms of the offer. B Neither additional terms nor different terms are ever included in the contract. C Additional terms that do not materially alter the original terms of the offer will be included in the contract, but different terms may be knocked out of the contract. D Additional terms may be knocked out of the contract, but different terms will be included unless the offeror has already objected to the particular terms.

Additional terms that do not materially alter the original terms of the offer will be included in the contract. In a contract between merchants for the sale of goods, additional terms in the acceptance will be included in the contract unless they materially alter the original terms, the offer expressly limits acceptance to the terms of the offer, or the offeror has already objected to the particular terms or objects within a reasonable time. Different terms may be knocked out of the contract. There is a split of authority on different terms. Some states follow the same rules as for additional terms, but others employ the knockout rule. Under the knockout rule, conflicting terms are knocked out of the contract and gaps are filled by the U.C.C. gap-filler provisions. Conflicting terms are subject to the knockout rule; additional terms are never subject to the knockout rule.

An advertisement, catalog, or circular letter, listing the price at which a seller is willing to sell a product, would typically be construed as: A An offer for a bilateral contract B An invitation for an offer C A merchant's firm offer D An offer for a unilateral contract

An advertisement, catalog, or circular letter, listing the price at which a seller is willing to sell a product, would typically be construed as an example of an invitation for an offer, rather than an offer. Since there is no clearly identified offeree, there is no offer of any type, whether for a bilateral contract, consisting of the exchange of mutual promises, a unilateral contract, where the offeror requests performance rather than a promise, or any type of merchant's firm offer.

Which one of the following elements is needed for a discharge of a contract due to frustration? A An unanticipated or extraordinary act or event has made the contractual duties impossible or impracticable to perform B A subsequently enacted law or other governmental act has made the subject of matter of the contract illegal C An act of nature has destroyed the contract's subject matter or the designated means for performing the contract D An unforeseen act or event has completely or almost completely destroyed the purpose of the contract

An element of frustration is that an unforeseen act or event has completely or almost completely destroyed the purpose of the contract. Frustration will exist if the purpose of the contract has become valueless by virtue of some supervening event not the fault of the party seeking discharge. If the purpose has been frustrated, a number of courts will discharge contractual duties even though performance of these duties is still possible. The elements necessary to establish frustration are: (i) some supervening act or event leading to the frustration; (ii) at the time of entering into the contract, the parties did not reasonably foresee the act or event occurring;(iii) the purpose of the contract has been completely or almost completely destroyed by this act or event; and (iv) the purpose of the contract was realized by both parties at the time of making the contract. A contract can be discharged by impossibility or impracticability when an unanticipated or extraordinary act or event has made the contractual duties impossible or impracticable to perform. Contractual duties can also be discharged by a subsequent act of nature that destroys the contract's subject matter or the designated means for performing the contract. But neither of these is considered discharge by frustration. A discharge by illegality occurs when the subject matter of the contract has become illegal due to a subsequently enacted law or other governmental act. This is often referred to as "supervening illegality."

An intended beneficiary must be: A Identifiable at the time performance is due B Present at the time the performance is due C Notified at the time the contract is made D Named at the time the contract is made

An intended third-party beneficiary has certain rights under the contract. The best test for determining whether someone is an intended beneficiary is to pose the following question: "To whom is performance to be given according to the language of the contract?" In other words, was the purpose of the promisee, according to the language of the contract, to get the benefit for herself primarily or to confer a right on another directly? If the purpose was to confer a right on another directly, there is a third-party beneficiary situation. The third-party beneficiary must be identifiable at the time performance is due. Although it is true that a court will more likely find that a contract is primarily for the benefit of a third party if that third party is expressly designated in the contract, this is not always the case. It is not necessary that the third-party beneficiary be named, or even identifiable, at the time the contract is made; she need only be identifiable at the time performance is due. There is no requirement that an intended beneficiary be notified at the time the contract is made or be present at the time the performance is due.

An offer for a bilateral contract can be accepted by: A Full performance only B Beginning performance only C A promise to perform only D A promise to perform or the beginning of performance

An offer for a bilateral contract may be accepted either by a promise to perform or by the beginning of performance. Note: Unless an offer specifically provides that it may be accepted only through performance, it will be construed as an offer to enter into a bilateral contract. In contrast, a unilateral contract can be accepted only by full performance . Note that the beginning of performance may create an option so that the offer is irrevocable. However, the offeree is not obligated to complete performance merely because he has begun performance, as only complete performance constitutes an acceptance of the offer.

Which of the following statements is correct? A Both an ordinary option contract and a merchant's firm offer require that the offeree give consideration B A merchant's firm offer requires that the offeree give consideration, whereas an ordinary option contract does not C An ordinary option contract requires that the offeree give consideration, whereas a merchant's firm offer does not D Neither an ordinary option contract nor a merchant's firm offer requires that the offeree give consideration

An ordinary option contract is a distinct contract in which the offeree gives consideration for a promise by the offeror not to revoke an outstanding offer. In contrast, under Article 2's merchant's firm offer provision, there are circumstances in which a promise to keep an offer open is enforceable even if no consideration has been paid to keep the offer open: A merchant's firm offer arises when a merchant offers to buy or sell goods in a signed writing and the writing gives assurances that the offer will be held open.

As a general rule, if the promisor fails to perform under a contract, a donee third-party beneficiary whose rights have vested can sue: A The promisee B The promisor C The promisee and the promisor D The promisee or the promisor

As a general rule, if a promisor fails to perform under a contract, a donee third-party beneficiary whose rights have vested can sue the promisor to enforce the contract. Absent detrimental reliance, a donee beneficiary cannot sue the promisee because generally there is no right to sue for nondelivery of a gift.

A man went to his local sporting goods store and told the salesperson that he wanted a tennis racket that was very high-end. The salesperson showed him a racket that he said was made of the finest titanium and would probably last for years. The man bought the racket and left the store. After playing with the racket for three days, it suddenly snapped in two when he hit a hard shot. He showed it to the tennis pro at his club who informed him that the racket was painted plastic. If the man sues the sporting goods store for a refund, under which theory or theories would he most likely prevail? A Breach of an express warranty that the racket was titanium. B Breach of an express warranty that the racket would last for years. C Breach of the implied warranty of merchantability. D Breach of both an express warranty that the racket was titanium and the implied warranty of merchantability.

Both express warranty and implied warranty theories are viable. An express warranty will arise from any statement of fact or promise. Here, the salesperson said that the racket was made of titanium and would last for years. The former is a statement of fact that will give rise to a warranty; thus, (A) is a good theory. The latter is not a statement of fact, but a prediction of the future. Moreover, it is not specific - how long is "years"? This statement amounts to mere puffery and will not give rise to a warranty. Therefore, (B) is not a good theory. An implied warranty of merchantability will arise in every sale by a merchant unless disclaimed. To be merchantable, goods must be fit for ordinary purposes, and arguably a racket that dents right away because it is made of plastic is not fit for ordinary purposes. Therefore, (C) is a good theory, making (D) the correct choice.

A homeowner and a builder entered into a written contract to build a sauna in a spare room in the homeowner's home at a cost of $3,000. The contract contained a clause stating that the builder will not begin construction without prior approval of the plans by the homeowner's certified public accountant. The builder submitted his designs to both the homeowner and the accountant. The homeowner liked the plans, but the accountant did not and withheld his approval. The builder asked the homeowner whether she wanted him to submit new designs. The homeowner told the builder orally, "No! Your designs are great! My accountant is crazy! You go right ahead and construct the sauna." The builder constructed the sauna. The homeowner now refuses to pay the builder, citing the clause requiring approval by the accountant. If the builder sues the homeowner, the builder will recover: A The full contract price, because the accountant's approval was not a condition precedent for the contract to take effect. B The full contract price, because once the builder began building the sauna after speaking to the homeowner, the homeowner did nothing to stop the builder. C The reasonable value of the builder's services and materials, because otherwise the homeowner would be unjustly enriched. D Nothing, because the homeowner's oral statement will be excluded by the parol evidence rule.

By her statement to the builder, the homeowner waived the benefit of the condition requiring the accountant's approval of the design plans, and the builder detrimentally relied on the statement by building the sauna. Thus, there is a binding waiver of the condition. A condition is an event, other than the passage of time, the occurrence or nonoccurrence of which creates, limits, or extinguishes the absolute duty to perform in the other contracting party. The occurrence of a condition may be excused under a number of different circumstances. One such circumstance is where the party having the benefit of the condition indicates by words or conduct that she will not insist upon it. If a party indicates that she is waiving a condition before it happens, and the person affected detrimentally relies on it, a court will hold this to be a binding estoppel waiver. The promise to waive the condition may be retracted at any time before the other party has detrimentally changed his position. Here, the contract provided that the builder could not begin work without the accountant's prior approval. This approval was a condition that had to be met before the homeowner's duty to pay would arise. When the homeowner told the builder to commence working on the sauna, even though the accountant had withheld his approval, the homeowner was telling the builder that she was waiving the condition of the accountant's approval. The builder then acted in detrimental reliance on this statement by in fact starting and completing the building of the sauna. While the homeowner could have retracted her statement and reinstated the condition prior to the builder's detrimental reliance, she did nothing when the builder began working on the sauna. Under such circumstances, the homeowner made a binding waiver of the condition and will be estopped from asserting it. Thus, the builder is entitled to recover the full contract price. (A) is incorrect because, as discussed above, the accountant's approval was a condition precedent for the parties' contractual duties to arise. The builder's duty to build the sauna and the homeowner's duty to pay for it would not arise without the condition of the accountant's approval either being satisfied or being excused. (C) is incorrect because unjust enrichment is a quasi-contract alternative that the builder could utilize if he did not have a contract remedy. Here, however, the builder can recover the full contract price because the homeowner waived the condition and is estopped from retracting the waiver. (D) is incorrect because the parol evidence rule does not prohibit evidence of a subsequent modification of a written contract; the rule applies only to prior or contemporaneous expressions. Consequently, it may be shown that the parties altered the integrated writing after its making. The oral agreement between the homeowner and the builder described in the facts was made subsequent to the writing. Therefore, the parol evidence rule is inapplicable to this agreement.

Which of the following best describes the elements required for an effective assignment? A An adequate description of the right being assigned, consideration, and present words of assignment B A writing containing an adequate description of the right being assigned and present words of assignment C An adequate description of the right being assigned and present words of assignment D Consideration and a writing containing an adequate description of the right being assigned and present words of assignment

For an assignment to be effective, there must be an adequate description of the right being assigned. In addition, there is a requirement that the assignment is expressed in present words of assignment. This means that the assignor must manifest an intent to transfer his rights under the contract completely and immediately to the assignee. Whether such intent is present will be determined by looking to the terms of the transfer itself; i.e., the test is objective, not subjective. It is not necessary to use the word "assign"; any generally accepted words of transfer will suffice (e.g., "convey," "sell," "transfer," etc.). A writing is usually not required to have an effective assignment; an oral assignment is generally effective. Situations where an assignment must be in writing include wage assignments; assignments of an interest in land; assignments of choses in action worth more than $5,000; and assignments intended as security interests under Article 9 of the U.C.C. Consideration is not required; a gratuitous assignment is effective.

When should the nonbreaching party treat an otherwise minor breach as a material breach? A When the breach is part of a divisible contract B When the breach causes the nonbreaching party damages C When the breach is coupled with an anticipatory repudiation D When the breach relates to the timing of performanceq

If a minor breach is coupled with an anticipatory repudiation, the nonbreaching party may treat it as a material breach. Thus, the nonbreaching party may sue immediately for total damages and is permanently discharged from any duty of further performance. The courts hold that the nonbreaching party must not continue on with the contract, because to do so would be a failure to mitigate damages. There is no reason that a minor breach that is part of a divisible contract should be treated as a material breach. In fact, in a divisible contract, recovery is available for substantial performance of a divisible part even if there has been a material breach of the entire contract. Even a minor breach can cause the nonbreaching party damages. The effect of a minor breach is to provide a remedy for the immaterial breach to the aggrieved party. The aggrieved party is not relieved of her duty of performance under the contract by a minor breach, unlike in the case of a material breach. Unless the nature of the contract is such as to make performance on the exact day agreed upon of vital importance, or the contract by its terms provides that time is of the essence, a failure by a promisor to perform at the stated time will not be material. Thus, a minor breach that relates to the timing of performance generally should not be treated as a material breach

If an accord agreement is breached: A by the debtor, the creditor may sue on both the original contract and for breach of the accord agreement B by the debtor, the creditor may sue either on the original contract or for breach of the accord agreement C by the creditor by refusing to accept the performance agreed to in the accord, the debtor is entitled to punitive damages D by the creditor by suing on the original contract, the debtor may immediately sue for damages for breach of the accord agreement

If an accord agreement is breached by the debtor, the creditor may sue either on the original contract or for breach of the accord agreement, but not on both. If the accord agreement is breached by the creditor by suing on the original contract, the debtor may either: raise the accord agreement as an equitable defense and ask that the contract action be dismissed or wait until the creditor is successful in the action (i.e., until the debtor is damaged) and then bring an action at law for damages for breach of the accord contract. The debtor may not immediately sue for damages. If the accord agreement is breached by the creditor refusing to accept the performance agreed upon in the accord, the debtor may bring an action for breach of the accord agreement, but is not entitled to punitive damages. Punitive damages generally are not awarded in contract cases.

In a shipment contract, when goods are destroyed en route from the seller to the buyer, the risk of loss is borne by: A The buyer because the risk of loss passed to the buyer at the time of the contract under the doctrine of equitable conversion B The seller because the risk of loss does not pass to the buyer until the shipment is tendered to the buyer C The seller because it was the seller's responsibility to contract with the carrier D The buyer because the risk of loss passed to the buyer when the goods were delivered to the carrier

In a shipment contract, the risk of loss passes to the buyer when the goods are delivered to the carrier. Any loss incurred en route is borne by the buyer. Equitable conversion is a doctrine that applies only to the sale of land, not goods. The risk of loss does not pass to the buyer until the goods are tendered to the buyer under a destination contract. This is a shipment contract, which means the risk of loss passes to the buyer when the goods are delivered to the carrier. While it is the seller's responsibility to contract with the carrier, that does not affect the risk of loss rules.

In a suit for restitution, the measure of recovery is: A the amount necessary to buy a substitute performance B nothing, if the plaintiff is the breaching party C the value of the benefit conferred D the difference between what the plaintiff would have received if the contract had been properly performed and the value of what the plaintiff actually received

In a suit for restitution, the measure of recovery is the value of the benefit conferred. Restitution is based on preventing unjust enrichment when one has conferred a benefit on another without gratuitous intent. The value of the benefit conferred is usually measured by the benefit received by the defendant, but it may also be measured by the reasonable value of the work performed by the plaintiff. The amount necessary to buy a substitute performance is an expression of the measure of expectation damages, not restitution. The measure of recovery is not necessarily nothing if the plaintiff is the breaching party. Under some circumstances, a plaintiff may seek restitution even though the plaintiff is the party who breached. For example, a buyer who has paid part of the purchase price may recover some payments even if he is in breach. The difference between what the plaintiff would have received if the contract had been properly performed and the value of what the plaintiff actually received is also a formulation of compensatory, expectation damages rather than restitution. This measure does not address unjust enrichment.

Which of the following statements is correct regarding damages for a breach of a contract for the sale of goods? A Either a nonbreaching buyer or a nonbreaching seller may recover consequential damages, but only a buyer may recover incidental damages B Either a nonbreaching buyer or a nonbreaching seller may recover incidental damages, but only a buyer may recover consequential damages C Either a nonbreaching buyer or a nonbreaching seller may recover consequential damages, but only a seller may recover incidental damages D Either a nonbreaching buyer or a nonbreaching seller may recover incidental damages, but only a seller may recover consequential damages

In contracts for the sale of goods, compensatory damages may also include incidental damages. Either a nonbreaching buyer or a nonbreaching seller may recover incidental damages for a breach of a contract for the sale of goods. Incidental damages include expenses reasonably incurred by the buyer in inspection, receipt, transportation, care, and custody of goods rightfully rejected and other expenses reasonably incident to the seller's breach, and by the seller in storing, shipping, returning, and reselling the goods as a result of the buyer's breach. Consequential damages are special damages over and above standard expectation damages. These damages result from the nonbreaching party's particular circumstances and are recoverable only if a reasonable person would have foreseen them as a probable result of breach. Note that in contracts for the sale of goods, only a buyer may recover consequential damages

Which of the following normally would not be an exception to the preexisting legal duty rule? A A minor's ratification of a contract upon reaching the age of majority. B A compromise based on an honest dispute as to duty. C Payment of a smaller sum to settle an existing debt. D An acceleration of the performance of the duty.

In the case of an existing debt, payment by the debtor of a smaller sum than due will not be sufficient consideration for a promise by the creditor to discharge the debt. However, because courts are anxious to avoid the preexisting duty rule, payment of a smaller debt may be sufficient consideration if the payment was in any way different (e.g., stock instead of cash) or if the debt was honestly disputed. Almost any variation, such as accelerating performance, is considered adequate consideration. A promise to perform a voidable obligation (e.g., a minor's ratification of a contract upon reaching the age of majority) is also enforceable despite the absence of new consideration. If the scope of the legal duty owed is the subject of honest dispute, then a modifying agreement relating to it will ordinarily be given effect.

A contract entered into between an infant and an adult is: A voidable by either party B voidable by the adult but binding on the infant C voidable by the infant but binding on the adult D void

Infants (generally those younger than age 18) lack capacity to enter into a contract binding on themselves. Adults have the capacity to bind themselves under a contract. Thus, a contract entered into between an infant and an adult is voidable by the infant but binding on the adult.

An interior decorator asked a woodworker she met at a crafts fair to build a curly maple armoire. They entered into a written contract, with a contract price of $6,500 to be paid upon the decorator's receipt of the armoire. When the work was completed, the woodworker shipped the armoire to the decorator. After inspecting it, the decorator felt that it was not of the same high level of workmanship as she was expecting, given the other furniture that the woodworker had showcased at the fair, and a good faith dispute arose between the parties as to the workmanship. The decorator sent the woodworker a check for $4,000 marked "payment in full." The woodworker indorsed and cashed the check, then sued the decorator to recover the $2,500 balance. Most courts would hold that: A The woodworker's cashing of the check constituted an accord and satisfaction, discharging the decorator's duty to pay the balance. B The woodworker can recover the $2,500 balance from the decorator. C The woodworker is estopped to sue for the balance because he cashed the check knowing that it was being tendered in full settlement. D The woodworker's indorsing a check so marked constituted a written release, thereby discharging the contract.

Most courts would hold that there is a good faith dispute, and the check thus proposed an accord; the woodworker's act of cashing it is a satisfaction. A contract may be discharged by an accord and satisfaction. An accord is an agreement in which one party to an existing contract agrees to accept, in lieu of the performance that he is supposed to receive from the other party, some other, different performance. Satisfaction is the performance of the accord agreement. An accord and satisfaction generally may be accomplished by tender and acceptance of a check marked "payment in full" where there is a bona fide dispute as to the amount owed. Here, there is a good faith dispute between the parties as to the workmanship on the armoire. Therefore, the decorator's tender of the check marked "payment in full" and the woodworker's cashing of the check constituted an accord and satisfaction, discharging her duty to pay the balance. (B) is incorrect because the debt is unliquidated. Generally, payment of a smaller sum than due will not be sufficient consideration for a promise by the creditor to discharge the debt. However, the majority view is that payment of the smaller amount will suffice for an accord and satisfaction where there is a bona fide dispute as to the claim. As discussed above, because the parties had a good faith dispute about the workmanship on the armoire, the decorator's tender of the check and the woodworker's cashing of the check constituted an accord and satisfaction, which discharged the decorator's duty to pay the balance. (C) is incorrect because a promissory estoppel situation does not exist in that there was no change of position by the decorator based on any act or statement by the woodworker. Whenever a party to a contract indicates that she is waiving a condition before it is to happen or some performance before it is to be rendered, and the person addressed detrimentally relies upon the waiver, the courts will hold this to be a binding (estoppel) waiver. Here, there is no indication that the designer detrimentally changed position as a result of the woodworker's cashing of the check. Therefore, his act of cashing the check could not be considered an estoppel waiver. (D) is incorrect because the woodworker's indorsement is not sufficient to meet the writing requirement for a release. A release that will serve to discharge contractual duties is usually required to be in writing and supported by new consideration or promissory estoppel elements. While the good faith dispute between the woodworker and the decorator would meet the consideration requirement for a release, the indorsement does not show the kind of circumspection and deliberateness that the writing requirement was intended to ensure. Therefore, the better answer is that the acceptance of the check by the woodworker was a satisfaction, as discussed above, rather than a release.

Mutual mistake can be a defense to the formation of a contract if: A The mistake has any effect on the agreed-upon exchange B The adversely affected party bore the risk of the mistake C The mistake concerns the value of the subject matter of the contract D The mistake concerns a basic assumption on which the contract is made

Mutual mistake can be a defense to the formation of a contract if the mistake concerns a basic assumption on which the contract is made. Such a mistake renders the contract voidable by the adversely affected party if the mistake has a material effect on the agreed-upon exchange (not any effect on the agreed-upon exchange) and the party seeking avoidance did not assume the risk of the mistake. Thus the answer choice stating that the adversely affected party bore the risk of the mistake is incorrect. A mistaken assumption as to the value of the subject matter generally will not be remedied.

On March 1, the purchasing agent for a suburban school district faxed a "quotation request form" to a supplier of school furniture requesting an offer for the sale of 20 student chairs. The form was on school district letterhead and signed by the purchasing agent. It specified that the offer must be held open for four months and that the price term must be no higher than $30 per chair. The supplier telephoned the purchasing agent and told him that he would sell the school district 20 chairs at $20 per chair. He also agreed to hold the offer open for four months. The purchasing agent thanked the supplier for the offer and indicated that he would get back to him within that time period. On May 1, before the purchasing agent had responded to the supplier's offer or taken any action in reliance on it, the supplier faxed a letter to the purchasing agent stating that demand for student chairs had been higher than expected and that the offer was terminated. On May 2, the purchasing agent called the supplier, told him that the school district was treating his offer as still being open, and accepted it on its terms. Did the purchasing agent's call on May 2 create a legally enforceable contract with the supplier? A Yes, because the contract is for the sale of goods valued at less than $500. B Yes, because the school district accepted the offer within three months. C No, because the supplier did not sign the form specifying the length of time that the offer would be held open. D No, because a firm offer under the U.C.C. is not effective if its term is more than three months.

No contract was created because the supplier effectively revoked his offer. Under the U.C.C., an offer by a merchant to buy or sell goods in a signed writing that, by its terms, gives assurances that it will be held open is not revocable for lack of consideration during the time stated (not to exceed three months). If the term assuring that the offer will be held open is on a form supplied by the offeree, it must be separately signed by the offeror. Here, the school district supplied the form stating that the offer must be held open for four months. The supplier's verbal assent to that requirement was not sufficient to qualify as a firm offer under the U.C.C. Thus, he was free to revoke his offer. (A) is incorrect because the fact that a writing would not be required under the Statute of Frauds if a contract had been formed between the parties is irrelevant. A writing is required for a firm offer under the U.C.C. regardless of the value of the goods offered. (B) is incorrect because the school district lost its power of acceptance when the supplier revoked his offer, regardless of the fact that three months had not passed. As discussed above, the supplier's offer did not constitute a firm offer under the U.C.C. (D) is incorrect because the fact that the term of the firm offer was more than three months does not invalidate it. If the stated period extends beyond three months, the firm offer will stand, but it will only last for the three-month maximum.

Nonfulfillment of a condition: A will excuse a duty to perform that was subject to the condition B gives rise to liability for nonperformance C does not excuse the other party's duty to perform under the contract D will result in a breach of contract

Nonfulfillment of a condition normally will excuse a duty to perform that was subject to the condition. A condition is a provision the fulfillment of which creates or extinguishes a duty to perform under a contract; thus, nonfulfillment of a condition will excuse the other party's duty to perform under the contract. Nonfulfillment of a condition is not a breach of contract and does not give rise to liability for nonperformance.

Which of the following is not traditionally an element of the doctrine of promissory estoppel? A The reasonable expectation that the promise will induce action or forbearance B Valuable consideration on both sides of the bargain C Detrimental reliance D An action or forbearance that is in fact induced by a promise

Promissory estoppel is considered a substitute for consideration. Thus, consideration is not necessary if the facts indicate that the promisor should be estopped from not performing. A promise is enforceable if necessary to prevent injustice if: (i) the promisor should reasonably expect to induce action or forbearance; and (ii) such action or forbearance is in fact induced. Detrimental reliance is simply another way of describing the action or forbearance of the promisee (i.e., he relied on the promise to his detriment).

Which of the following is a key distinction between an anticipatory repudiation and a prospective failure to perform? A Repudiation must be unequivocal, whereas prospective failure to perform is determined by the subjective beliefs of the other party. B Repudiation is final, whereas prospective failure to perform may be retracted. C Repudiation must be unequivocal, whereas prospective failure to perform involves mere doubts. D Repudiation may be retracted, whereas prospective failure to perform is a breach and cannot be retracted.

Prospective inability or unwillingness to perform differs from anticipatory repudiation because repudiation must be unequivocal, whereas prospective failure to perform involves conduct or words that merely raise doubts that the party will perform. Repudiation must be unequivocal. However, a prospective failure to perform is not based on the subjective beliefs of the other party, but rather is judged on a reasonable person standard. Both repudiation and prospective failure to perform may be retracted, provided the other party has not yet changed position in reliance on the repudiation or prospective failure. The effect of a prospective failure is to allow the innocent party to suspend performance until she receives adequate assurances. She may treat this situation as a breach only if the assurances are not given. If a defaulting party regains his ability or willingness to perform, he must communicate that to the other party.

Which of the following promises is commonly considered to be illusory? A A promise with an unqualified right to cancel or withdraw at any time B A promise conditioned on the promisor's satisfaction C A promise to purchase all that one requires D A promise to sell all that one decides to make

Reservation of an unqualified right to cancel or withdraw at any time would be considered an illusory promise. "Requirements" contracts (i.e., promises to purchase all that one requires ) and "output" contracts (i.e., promises to sell all that one decides to make) are enforceable, as the promisor has parted with the legal right to buy (or sell) the goods he may need (or make) from (or to) another source. A promise conditioned on the promisor's satisfaction is not illusory because the promisor is constrained by good faith (for contracts involving personal taste) and a reasonable person standard (for contracts involving mechanical fitness, utility, or marketability).

Which of the following statements is true regarding a specific performance remedy for breach of a contract to provide services? A Specific performance is not available as a remedy for a breach of a contract to provide services B Specific performance can always be granted for a breach of a contract to provide services because services are personal and thus always considered to be rare or unique C Specific performance can be granted for a breach of a contract to provide services only if a legal remedy would be inadequate D Specific performance can be granted for a breach of a contract to provide services only if the services are shown to be rare or unique

Specific performance is not available for breach of a contract to provide services, even if the services are rare or unique and a legal remedy would be inadequate. This is because of problems of enforcement (it would be difficult for the court to supervise the performance) and because the courts feel it is tantamount to involuntary servitude, which is prohibited by the Constitution. Generally a court may grant specific performance, which is essentially an order from the court to the breaching party to perform or face contempt of court charges, if the legal remedy is inadequate. The legal remedy (damages) generally is inadequate when the subject matter of the contract is rare or unique. The rationale is that if the subject matter is rare or unique, damages will not put the nonbreaching party in as good a position as performance would have, because even with the damages the nonbreaching party would not be able to purchase substitute performance. A contract to provide services is an exception to this general rule for the reasons stated above.

A large wholesale dealer in produce had never done business with a certain greengrocer who operated a small chain of markets in the Midwest. They entered into a written agreement whereby the wholesale dealer agreed to supply to the greengrocer the "fuzzy" variety of peaches at $35 per 50 pound lot. The agreement contained a provision stating that the greengrocer will buy "as many 50 pound lots of fuzzy peaches as the greengrocer chooses to order." Assuming that the greengrocer has not yet placed any orders for peaches with the wholesale dealer, is this agreement between the parties enforceable? A Yes, because it is a valid requirements contract and, as such, is enforceable under the Uniform Commercial Code. B Yes, because the Uniform Commercial Code will imply reasonable terms. C No, because the total quantity of the contract is not specified. D No, because there is no consideration on the greengrocer's part.

The agreement is not enforceable because the greengrocer's promise is illusory. For a contract to be enforceable, consideration must exist on both sides, i.e., each party's promise must create a binding obligation. If one party has become bound but the other has not, the agreement lacks mutuality because one of the promises is illusory. Here, the wholesale dealer has promised to supply the greengrocer with fuzzy peaches at a fixed price. The greengrocer, however, has not promised to order any peaches from the wholesale dealer. Even if the greengrocer decides to sell fuzzy peaches, it has not bound itself to order them from this particular wholesale dealer. The illusory nature of the greengrocer's promise makes the agreement unenforceable on consideration grounds. (A) is incorrect because in a valid requirements contract, both parties' promises create binding obligations: The promisor binds itself to buy from the supplier all that it requires, and the supplier binds itself to sell to the promisor that same amount. Consideration exists because the promisor is suffering a legal detriment; it has parted with the legal right to buy the goods it may need from another source. Under the U.C.C., which governs in this case because a contract for the sale of goods is involved, a good faith term is implied: The buyer's requirements means such actual requirements as may occur in good faith. Thus, if the provision had stated instead that the greengrocer will buy "as many 50 pound lots of fuzzy peaches as the greengrocer shall require," it would be a valid requirements contract under the U.C.C. because it requires the greengrocer to buy fuzzy peaches only from the wholesale dealer and to act in good faith in setting its requirements. (B) is incorrect even though the U.C.C. will imply reasonable terms under certain circumstances. Such terms as price and time for performance need not be spelled out in the contract; the terms will be supplied by a "reasonableness" standard if that is otherwise consistent with the parties' intent. However, supplying reasonable terms will not change the express terms of the contract. The provision that the greengrocer will buy as many peaches as it chooses to order is not sufficiently obligatory to be saved by the court supplying reasonable terms. (C) is incorrect because if the agreement were otherwise a valid requirements contract, the absence of a total quantity term would not matter. As a general rule in sale of goods contracts, the quantity being offered must be certain or capable of being made certain. The U.C.C. provides that an agreement to buy all of one's requirements is sufficiently certain because requirements usually can be objectively determined. Furthermore, the quantity ultimately required in good faith must not be unreasonably disproportionate to any stated estimate or any normal requirements (in the absence of a stated estimate). Hence, if the greengrocer had contracted to buy all of its requirements from the wholesale dealer, the absence of a term specifying total quantity would not have made the agreement unenforceable.

Under Article 2, when an offeree proposes additional or different terms during acceptance, what will the court apply to determine whether the additional or different terms become part of the contract? A The battle of the forms provision B The mailbox rule C Gap fillers D The mirror image rule

The battle of the forms provision of Article 2 lists specific rules for determining what terms are included in a contract when the terms of acceptance do not match the terms of the offer. Article 2 has abandoned the mirror image rule, which requires an absolute and unequivocal acceptance of each and every term of the offer. Gap fillers are used when certain terms are not included in the contract; it does not apply to additional or different terms in the acceptance. The mailbox rule is applied to determine the timing of acceptance of a contract.

After extensive negotiations, representatives for an automobile manufacturer and a tire maker orally agreed on the specifications for a supply of tires. The lawyer for the car manufacturer then drafted a written agreement and sent it to the tire maker's lawyer, who modified the draft and sent it back to the car manufacturer. This writing was signed by both parties. The tire maker now brings an action for breach of contract against the car manufacturer seeking damages. The car manufacturer attempts to introduce the testimony of its chief negotiator describing the oral agreement with the tire maker representatives that the tires would meet certain requirements of the car models. The tire maker objects, arguing that the parol evidence rule bars admission of this testimony. Which of the following is the best argument supporting admission of the testimony? A The memorandum signed by the parties was not a complete integration of their agreement. B The parol evidence rule does not bar evidence interpreting a written agreement. C The car manufacturer detrimentally relied on the oral agreement in signing the memorandum. D The parol evidence rule does not exclude misrepresentations.

The best argument for admission of the testimony is that the memorandum does not cover the entire agreement between the parties and was thus not a complete integration. Because the writing contains no mention of the oral agreement to meet certain car models requirements, the testimony would not "interpret" it in any way. Thus, (B) is incorrect. (C) and (D) are wrong because there is no evidence that the car manufacturer detrimentally relied on the oral agreement in signing the memorandum, or that the tire supplier's promise constituted a misrepresentation at the time it was made.

A buyer agreed to buy a limited edition guitar from a seller for $12,000 and a contract memorializing the agreement was signed by both parties. The next day, after the seller received an offer of $20,000 for the guitar, he called the buyer and said that he could not sell the guitar to him for $12,000. The buyer did not respond. On the delivery date, the buyer fails to tender $12,000 and the seller does not deliver the guitar. On these facts: A The seller can recover from the buyer for breach of contract. B The buyer can recover from the seller for breach of contract. C Neither the seller nor buyer can recover until one of the parties tenders performance. D The contract is terminated.

The buyer can recover because the seller's phone call was an anticipatory repudiation. Anticipatory repudiation occurs where a promisor, prior to the performance time, unequivocally indicates that he cannot or will not timely perform, allowing the nonrepudiator the option of suspending performance and waiting to sue until the performance date, or to sue immediately. The seller's phone call was an unequivocal statement that he would not sell the guitar for $12,000. This repudiation excused the buyer's duty to tender $12,000 on the delivery date. (A) is incorrect because the seller's anticipatory breach excused the buyer's performance. (C) is incorrect because the seller's repudiation the day after the contract was signed gave rise to an immediate cause of action. (D) is wrong because the contract was not terminated; rather, it was anticipatorily breached by the seller.

On January 1, a car salesman offered to sell an antique car to a collector for $35,000 cash on delivery. The collector paid the car salesman $100 to hold the offer open for a period of 25 days. On January 4, the collector called the car salesman and left a message on his answering machine, asking him whether he would consider lowering the price to $30,000. The car salesman played back the message the same day but did not reply. On January 9, the collector wrote the car salesman a letter, telling him that he could not pay more than $30,000 for the antique car, and that if the car salesman would not accept that amount, he would not go through with the deal. The car salesman received this letter on January 10 and again did not reply. The car salesman never heard from the collector again. When did the offer that the car salesman made to the collector on January 1 terminate? A On January 4, when the collector made a counteroffer. B On January 9, when the collector mailed to the car salesman what amounted to a rejection. C On January 10, when the car salesman received from the collector what amounted to a rejection. D On January 25, when the 25-day option expired.

The car salesman's offer terminated on January 25, when the 25-day option expired. An option is a distinct contract in which the offeree gives consideration for a promise by the offeror not to revoke an outstanding offer. The collector paid the car salesman $100 to hold the offer open for a period of 25 days, and the offer could not be terminated before that time, not even by the offeree (here the collector). Nor did the offer survive the option period because the option specifically identified how long the offer would be open. (A) is incorrect because the collector's words did not amount to a counteroffer because they merely inquired as to whether the collector would consider lowering his price; the words were not unequivocal. (Even if this were a counteroffer, it would not extinguish the car salesman's offer, because of the option, as explained above.) (B) is incorrect because, as discussed above, even a rejection by the offeree will not terminate the option. Also, if the communication were effective as a rejection, it would be effective when received by the offeror. (C) is incorrect because, as discussed above, even unequivocal words of rejection by the offeree will not extinguish an option, absent detrimental reliance on the part of the offeror, which was not the case here.

A small computer company contracted with a community college to supply computer tech support for all of the college's faculty and staff beginning September 1. Two weeks before the date that performance was to begin, the computer company received a more lucrative offer at a large university. In order to fulfill its contract obligations to the community college, the company delegated its duties under the contract to another computer company for $1,000 above the contract price. The community college agreed to allow the replacement computer company to perform the services, but on September 1, the replacement company refused to perform. If the community college sues the replacement company, what will the result be? A The replacement company will prevail, because the community college's only remedy is against the computer company. B The replacement company will prevail, because $1,000 is grossly inadequate consideration. C The community college will prevail, because there has been a novation. D The community college will prevail, because it is an intended third-party beneficiary of the computer company replacement company contract, and the replacement company assumed the duty of performance under the contract.

The community college will prevail in its suit against the replacement company because there was a delegation of duties plus an assumption of duty on the part of the replacement company, supported by consideration, creating a third-party beneficiary situation in which the obligee can compel performance or bring suit for nonperformance. Because the replacement company has promised that it will perform the duty delegated and that promise is supported by consideration, an assumption of duty has occurred and a third-party beneficiary situation is created in which the community college can compel performance or bring suit for nonperformance. Thus, (D) is correct and (A) is incorrect. (B) is incorrect because a court would probably not even inquire into adequacy of consideration when the value of the consideration was satisfactory to the parties involved. (C) is incorrect because the facts do not clearly indicate a novation. A novation occurs where a new contract substitutes a new party to receive benefits and assume duties that had originally belonged to one of the original parties under the terms of the old contract. A novation discharges the old contract. A novation will be found when there is (i) a previous valid contract; (ii) an agreement among the parties, including the new party to the new contract; (iii) the immediate extinguishment of contractual duties as between the original contracting parties; and (iv) a valid and enforceable new contract. Here, the facts do not state or otherwise indicate that a new contract was created and the duties under the old contract were discharged. Thus, this is not a novation situation.

When a court refuses to enforce a provision of a contract to avoid "unfair" terms, it is applying the concept of: A disaffirmance B pari delicto C promissory estoppel D unconscionability

The concept of unconscionability allows a court to refuse to enforce a provision of a contract to avoid unfair terms. There are two types of unconscionability: substantive unconscionability based on lopsided terms, and procedural unconscionability based on unfair surprise or unequal bargaining power. A party might be able to successfully seek relief from a contract based on a defense of illegality, if the party is not in pari delicto, i.e., as culpable as the other party to the illegal contract. An infant might apply the concept of disaffirmance by choosing to disaffirm a contract he entered into before reaching the age of majority. Promissory estoppel allows a promise, unenforceable due to a lack of consideration, to be enforced if necessary to prevent injustice.

The manager of the shoe department of a large department store noticed that mukluks were flying off the shelf in anticipation of another exceptionally cold winter, and he realized that he needed to order more. On November 1, the manager phoned a local manufacturer of mukluks and placed an order for 100 pairs, at a cost of $90 a pair, the price listed in the manufacturer's catalogue, which the manager had consulted before placing his order. Two days later, the manufacturer mailed the manager a letter stating that the mukluks were now $105 a pair and that they would be shipped to him on November 17. The manager received the letter on November 5, but he never responded. On November 17, the manufacturer shipped the mukluks to the department store, but it was not a perfect tender, and the manager filed suit for breach of contract. Assuming that the parties' communications were sufficient to form a contract, on what day was the contract formed? A November 1, the day the manager placed his order. B November 3, the day the manufacturer sent its letter. C November 5, the day the manager received the letter. D November 17, the day the mukluks were shipped.

The contract was formed on November 3. An offer to buy goods for shipment is generally construed as inviting acceptance either by a promise to ship or by shipment. Here, the letter constitutes a promise to ship and thus is an acceptance. The rule for acceptances is that they are effective as soon as they are dispatched, which was November 3. Thus, (B) is correct, and (C) is wrong. (A) is wrong because the order was an offer, not an acceptance to the catalogue. (D) is wrong because acceptance occurred before shipment when the manufacturer sent its promise to ship.

The owner of a summer house entered into a written agreement with a plumber. The contract contained a clause requiring all plumbing work to be completed by noon on June 1 and provided that the homeowner would pay the plumber $1,200 for his work. The plumber began working on the job on May 28. When he quit working for the day on the afternoon of May 29, half of the job was completed. Shortly thereafter, a heavy rain began which caused a flash flood the next day, which swept the house away. Which of the following best describes the obligations of the plumber and the homeowner after the flood? A Neither the plumber nor the homeowner is discharged from their obligations under the contract. B The homeowner is obliged to pay the plumber $1,200. C The plumber is discharged from his obligation but is entitled to recover from the homeowner the fair value of the work he performed prior to the flood. D Neither the plumber nor the homeowner have any further obligations.

The destruction of the house discharges the plumber's duties due to impossibility, but the plumber has a right to recover for the reasonable value of the work he performed. Contractual duties are discharged where it has become impossible to perform them. The occurrence of an unanticipated or extraordinary event may make contractual duties impossible to perform. If the nonoccurrence of the event was a basic assumption of the parties in making the contract, and neither party has assumed the risk of the event's occurrence, duties under the contract may be discharged. Impossibility must arise after entering into the contract. If there is impossibility, each party is excused from duties that are yet to be performed. If either party has partially performed prior to the existence of facts resulting in impossibility, that party has a right to recover in quasi-contract at the contract rate, or for the reasonable value of his performance if that mode of valuation is more convenient. While that value is usually based on the benefit received by the defendant (unjust enrichment), it also may be measured by the detriment suffered by the plaintiff (the reasonable value of the work performed). Here, the house on which the plumber was to perform plumbing repairs was totally destroyed in a flood. The facts indicate that this flood was of such an unexpected nature that its nonoccurrence was a basic assumption of the parties, and neither party was likely to have assumed the risk of its occurrence. Thus, it has become literally impossible for the plumber (or anyone else) to complete the job. This impossibility will discharge both the homeowner and the plumber from performing any contractual duties still to be fulfilled. Therefore, the plumber need not finish the repair work, and the homeowner is not obligated to pay the entire amount of $1,200. However, the plumber can recover under quasi-contract. (A) and (B) are therefore incorrect for these same reasons. (D) is incorrect because it fails to account for the fact that the homeowner will have to pay the plumber for the value of the work already performed.

On November 5, an athletic shoe manufacturer entered into a written contract with a shoe store to carry its running shoes. The contract included a provision that "5% of the proceeds attributable to the sale of the manufacturer's shoes by the shoe store during the month of February (American Heart Month) each year would be donated to the local hospital's new cardiovascular wing." The day after the parties signed their contract, the store owner informed the hospital of the planned donation and indicated that the hospital could expect to receive about $1,500 in early March. In anticipation of the donation, the hospital purchased a new heart monitor on January 5. On January 15, because the Christmas sales season had been poor, the manufacturer and the shoe store agreed to modify their contract to eliminate the provision for payments to the hospital's cardiovascular wing. What effect would this subsequent modification have on the hospital's right to institute an action for the lost proceeds (assuming that it otherwise had standing to collect the same)? A It would have no effect, because the hospital's rights vested when it learned of the original agreement. B It would have no effect, because the hospital detrimentally relied on the parties' agreement before finding out that they had changed the agreement. C It would have no effect, because the hospital's rights vested when the contract was made. D It would have the effect of cutting off the hospital's right to institute an action under the original agreement.

The hospital can still sue because it detrimentally relied on the agreement. An intended third-party beneficiary can enforce a contract only after its rights have vested. Vesting occurs when the beneficiary: (i) manifests assent to the promise in a manner invited or requested by the parties; (ii) brings suit to enforce the promise; or (iii) materially changes position in justifiable reliance on the promise. Here, the hospital purchased the heart monitor in detrimental reliance on the agreement between the manufacturer and the shoe store. The reliance was reasonable because the store owner informed the hospital of the agreement, the approximate amount that they would receive, and the date they would receive it. (A) and (C) are incorrect because, as noted above, the third-party beneficiary's rights do not vest until occurrence of one of the specified conditions. Vesting does not automatically occur on execution of the contract or when the beneficiary learns of the promise. (D) is incorrect because the hospital's right to institute an action under the original agreement could not be cut off once the right vested, and it vested when the hospital purchased the heart monitor before the parties executed their new agreement.

The father of a young child whose life was saved by doctors at a hospital emergency room called the president of the hospital and told her that he will donate $50,000 to the hospital, payable in 90 days, in consideration of the doctors' saving his child's life. The president recognized the father's name as a wealthy philanthropist who had donated generously to other worthy causes, and accepted the offer. The president promptly informed the hospital's board of directors of the father's offer and her acceptance of the offer. The board approved the purchase of a new respirator for $50,000 and authorized the president to enter into a written contract with a medical devices company to make the purchase, which she did. Once the child recovered, the father changed his mind and refused to give the hospital the $50,000. If the hospital sues the father, what would the hospital's best theory be to recover the $50,000? A The father's offer was supported by consideration. B The father was estopped from not performing under a theory of detrimental reliance. C The father's offer constituted fraud in the inducement. D The terms of the father's offer were definite and certain.

The hospital's best theory to recover the $50,000 would be promissory estoppel (detrimental reliance). Under section 90 of the Second Restatement, a promise is enforceable, even without consideration, if the promisor reasonably expects to induce action or forbearance and such action or forbearance is in fact induced. (The remedy may be limited as justice requires.) Here, the father called the hospital's president and promised to donate $50,000 to the hospital. Based on that promise, and the president's knowledge that the father was a wealthy philanthropist who could make good on the promise and had donated generously to other worthy causes, the hospital's board of directors authorized the hospital president to purchase a new respirator for $50,000, which she did. The father should have reasonably expected to induce such action, and such action was in fact induced. Therefore, the father was estopped from not performing. (A) is incorrect because in most jurisdictions "past" or "moral" consideration is not sufficient consideration because it was not bargained for. If something was already given or performed before the promise was made, it will not satisfy the "bargain" requirement because it was not given in exchange for the promise. Here, the doctors had already saved the daughter's life when the father made his offer; the father's promise was not made to induce the doctors to act, nor did the doctors act to induce the father's promise. Thus, (A) is not correct. (C) is not correct because there is nothing in the facts to suggest that the father engaged in fraud in the inducement, i.e., that he made his promise knowing he had no intention of keeping it; rather, he simply changed his mind. Fraud in the inducement occurs when one party induces another to enter into a contract by asserting information he knows to be untrue, and the innocent party justifiably relies on the fraudulent misrepresentation. In such cases, the contract is voidable by the innocent party. (D) is incorrect because, although it is true that the father's offer was definite and certain, there can still be no contract absent consideration.

Which of the following would not be considered valuable consideration that supports a contract? A A benefit with no economic value. B Peace of mind for the promisor. C The gratification of influencing the mind of another. D Fulfillment of a condition to receive a gift.

The mere fulfillment of a condition to receive a gift is not adequate consideration. The fulfillment of the condition must be of some benefit to the promisor to constitute proper consideration. The benefit to the promisor need not have economic value. Peace of mind or the gratification of influencing the mind of another may be sufficient to establish bargained-for consideration.

An amateur sculptor ordered from an art museum's gift shop a miniature reproduction of a favorite statue, which the museum's flyer indicated was for sale only at the gift shop. He sent the museum an order form printed from its website and a check to cover the cost of the statue. The museum received and processed his order, but by mistake sent, by common carrier, a reproduction of a different statue. The statue was stolen from the carrier en route to the sculptor's home. Between the buyer and the museum, who bears the loss? A The loss falls on the museum because it was the museum that selected the carrier and made the arrangements for shipment. B The loss falls on the museum because it sent the wrong statue. C The loss falls on the buyer because the statue was advertised as being available for sale only at the museum's gift store, and therefore the risk of loss shifted to the buyer. D The loss falls on the buyer because title to the statue passed to him upon shipment, and he therefore bears the risk of loss.

The museum suffers the loss because the risk of loss was still with it when the statue was stolen. Under the U.C.C., when the contract authorizes or requires the seller to ship the goods by carrier but does not explicitly require the seller to deliver them at a particular location, risk of loss passes to the buyer when the goods are duly delivered to the carrier. [U.C.C. §2-509] However, if the buyer has a right to reject the goods, the risk of loss does not pass until the defects are cured or the buyer accepts the goods. [U.C.C. §2-510(1)] Here, the shipment of a nonconforming good constituted a breach, and the buyer had a right to reject it. Thus, the loss falls on the museum with respect to the theft. (A) is incorrect because risk of loss here is determined by the fact that the shipment was a breach of contract; it is not determined by who selected the carrier and made the arrangements. (C) is incorrect because it is irrelevant what the museum's advertisements say, since they are not part of the contract but rather are solicitations for offers. (D) is incorrect because, as stated above, risk of loss does not pass to the buyer if the good is so defective that the buyer has a right to reject it. (D) would be correct if not for the breach, because this is probably a shipment contract (requiring the museum to put the goods in the hands of a carrier, but not requiring delivery at a particular destination).

A man of seemingly modest means died, leaving his nephew as his sole heir. Among the items inherited by the nephew were some old oil paintings. The nephew knew nothing about art and had no place to put the paintings in his home. He placed an ad in the paper offering to sell the paintings at a price to be mutually agreed upon. A buyer for an art gallery responded to the ad. The buyer did not identify himself as an art gallery buyer or tell the nephew that he was knowledgeable about art. Rather, he concocted a story about wanting the paintings for his country estate. The nephew, for his part, revealed his lack of knowledge about art when he told the buyer that his uncle had probably painted the pieces himself. From the signature and the style, the buyer recognized that the artist was a renowned 19th century American portrait artist. The nephew and the buyer agreed upon a price and executed a contract. However, before the nephew delivered the paintings to the buyer, or the buyer paid him, he sought to rescind the contract. The buyer insisted that the nephew deliver the paintings to him and threatened to sue for breach of contract if he did not. Which argument would give the nephew the best basis for rescinding the contract with the buyer? A The nephew told the buyer that his uncle had probably painted the paintings himself. B The nephew did not know that the buyer was a professional buyer for an art gallery and was knowledgeable about art. C The buyer falsely told the nephew that the paintings were going to be used to furnish his (the buyer's) country estate. D The contract was still executory on both sides.

The nephew may be able to rescind the contract on the grounds of unilateral mistake if the buyer was aware that the nephew was mistaken about the identity of the artist. Where only one of the parties is mistaken about facts relating to the agreement, the mistake usually will not prevent formation of the contract. However, if the nonmistaken party is aware of the mistake made by the other party, he will not be permitted to snap up the offer; i.e., the mistaken party will have the right to rescind the agreement. Under the facts in this choice, the buyer knows that the nephew is mistaken about the identity of the artist, which is a basic assumption of the contract for the paintings. To obtain rescission, the nephew would also have to establish that the mistake creates a material imbalance in the exchange and that he did not assume the risk of that mistake. The facts in choice (A) give him the best grounds for doing so. (B) is incorrect because the fact that one of the parties to the contract has superior knowledge about the subject matter of the contract does not by itself justify rescission, even if the other party is unaware of that fact. The buyer's knowledge or lack of it was not a basic assumption on which the contract was made and was not relied on by the nephew in making the sale. (C) is incorrect because the buyer's misrepresentation to the nephew as to how he will use the paintings does not appear to have been relied on by the nephew. Hence, the misrepresentation is not significant enough to serve as grounds for rescinding the contract. (D) is incorrect because while it is true that a contract must be executory on both sides to be effectively discharged by rescission, this fact alone will not be sufficient to effect a rescission. Rather, when only one of the parties is seeking rescission, as is the case here, that party must prove an adequate legal ground (e.g., mistake, misrepresentation, duress, and failure of consideration). In this case, as discussed above, the ground of unilateral mistake will provide the nephew with the best basis for rescinding the contract.

Which of the following types of evidence may be outside the scope of the parol evidence rule? A Evidence of a condition precedent to effectiveness B Evidence of a contemporaneous oral agreement C Evidence of a prior oral agreement D Evidence of a condition precedent to performance

The parol evidence rule prohibits admissibility of extrinsic evidence that seeks to vary, contradict, or add to an integration. Other forms of extrinsic evidence may be admitted when they will not bring about this result, i.e., they will fall outside the scope of the parol evidence rule. When a party asserts that there was an oral agreement that the written contract would not become effective until a condition occurred, all evidence of the understanding may be offered and received. This would be a condition precedent to effectiveness. The rationale is that one is not altering a written agreement by means of parol evidence if the written agreement never came into being. It should be borne in mind that parol evidence of such a condition precedent will not be admitted if it contradicts the express language of the written contract. Parol evidence is inadmissible as to conditions precedent to performance, i.e., an oral agreement that the party would not be obliged to perform until the happening of an event. This latter type of condition limits or modifies a duty under an existing or formed contract. Under the parol evidence rule, when the parties to a contract express their agreement in a writing with the intent that it embody the final expression of their bargain (i.e., the writing is an integration), any other expressions, written or oral, made prior to the writing, as well as any oral expressions contemporaneous with the writing, are inadmissible to vary the terms of the writing.

A print cartridge company sent a letter to a business office offering to supply a free premium printer if the business office would agree to purchase all the print cartridges the office would need from the print cartridge company. This letter arrived in the hands of the business owner on the same day the office printer failed. The business office had experienced a slow month and the business owner was debating on whether to pay the office rent for the month or fix the printer. Based on the offer, the owner paid the rent. A week after she put the rent check in the mail, the owner received a second letter from the print cartridge company indicating that the printer program was being canceled due to a lack of printers. The next day, before she read the second letter, the owner mailed her acceptance letter to the print cartridge company. The print cartridge company refused to supply the business owner with a printer. If the business owner brings a breach of contract action against the print cartridge company, what result? A Judgment for the print cartridge company, because its offer to the business office was not sufficiently definite. B Judgment for the print cartridge company, because its revocation letter was received by the business owner before she dispatched the acceptance letter. C Judgment for the business owner, because she mailed her acceptance letter without being aware that the print cartridge company had revoked its offer. D Judgment for the business owner, because the owner used funds to pay the office rent in reliance on the print cartridge company's offer.

The print cartridge company will prevail. A revocation generally is effective when received by the offeree. Hence, the business owner's receipt of the revocation letter before she dispatched her acceptance letter effectively revoked the offer (even though the business owner was unaware of its contents when she mailed the acceptance). (A) is wrong because an offer to make a requirements contract (i.e., the buyer promises to buy from a certain seller all the goods it requires and the seller agrees to sell that amount to the buyer) is sufficiently definite because the quantity is capable of being made certain by reference to objective, extrinsic facts (i.e., the buyer's actual requirements). (C) is wrong because, as stated above, the revocation was effective when it was received (i.e., when it came into the offeree's possession), not when the offeree becomes aware of the revocation. (D) is wrong because the business owner's response to the offer was not reasonably foreseeable. As an exception to the general rule that a revocation is effective on receipt, an offer cannot be revoked and will be treated as an option contract for a reasonable length of time where the offeror could reasonably expect that the offeree would rely to her detriment on the offer. However, this exception usually is applied in only two circumstances: an offer for a unilateral contract and a subcontractor's bid to a general contractor. If the offeror is seeking a bilateral contract in a circumstance other than that of a subcontractor, it would be extremely rare for the offer to be irrevocable due to detrimental reliance. Generally, an offeree must accept the offer before relying on it. Here, it is not even clear that the business owner suffered a detriment (unless she could have skipped paying the rent that month). Even if that were the case, nothing suggests that the cartridge company reasonably should have foreseen that the business owner would rely on the offer the way she did (without having accepted it yet).

The rights of a third-party beneficiary vest when he: A Signs the contract; makes an enforceable promise in the contract; or provides consideration for the promise in the contract B Is named in the contract; is identifiable by the terms of the contract; or is identifiable at the time of performance C Receives notice of the existence of the contract; is a creditor of the promisee; or is an intended, not incidental, beneficiary D Manifests assent to the promise; brings suit to enforce the promise; or materially changes position in justifiable reliance on the promise

The promisor and promisee are generally free to modify the contract, and need not consult the third-party beneficiary unless his rights have vested. Once the third-party beneficiary's rights have vested, the promisor and promisee cannot vary his rights without his consent. A third-party beneficiary's rights vest when the beneficiary: manifests assent to the promise in a manner invited or requested by the parties; brings suit to enforce the promise; or materially changes position in justifiable reliance on the promise. In determining the promisee's intentions in a third-party beneficiary situation, courts will often look at whether the third party is expressly designated in the contract. If so, it is more likely that it is primarily for his benefit. But it is not necessary that the third-party beneficiary be named, or even identifiable, at the time the contract is made; he need only be identifiable at the time performance is due. Even if a third party is named or is otherwise identifiable, the promisor and promisee are free to modify the contract if the third party's rights have not yet vested as discussed above. A party that signs the contract, makes an enforceable promise in the contract, or provides consideration for the promise in the contract would most likely be considered a party to the contract, not a third-party beneficiary to the contact. In any event, these are not the factors that cause a third-party beneficiary's rights to vest. While it is true that only intended, not incidental, beneficiaries have rights under the contract, these rights must be vested to be enforceable. It is not enough that the third party has received notice of the existence of the contract. If his rights have not yet vested, as described above, the original parties are free to take actions, such as rescission or modification, which may affect the third-party beneficiary. These vesting rules apply to both creditor and donee beneficiaries.

On March 1, a health food store owner and a health food distributor entered into a written agreement providing that the distributor would supply the store owner with his natural foods requirements over the next 12 months. In return, the store owner agreed to purchase only from the distributor. The agreement also provided that payment for any purchases made under the agreement during the month of April would be turned over to a nonprofit food research corporation, "to carry on its good works." On April 1, the store owner ordered and the distributor shipped 20 cases of health food products, at a wholesale price of $4,000, which remains unpaid. On April 15, the store owner sold his store, inventory, and accounts receivable to a chain operation that also sells health food products. As part of the sale, the store owner assigned to the chain operation the contract with the distributor. The chain operation promptly notified the distributor of the sale and assignment. The $4,000 for the April 1 transaction remained unpaid and, on May 31, the food research corporation commenced suit to collect the money. Against whom may the action be maintained? A The store owner only. B Both the store owner and the chain operation. C The chain operation only. D Neither the store owner nor the chain operation.

The store owner remains liable based on his contract with the distributor, and the chain operation is liable based on its assumption of the contract with the distributor. The food research corporation is expressly designated in the contract between the store owner and the distributor as a party to whom payment for any April purchases is to be directly made. Thus, the portion of the contract providing for payment is primarily for its benefit. Consequently, the food research corporation is an intended third-party beneficiary of the store owner's promise to make the April payment. As the third-party beneficiary, it has a right of action against the promisor (the store owner) for enforcement of the promise to pay. When the store owner assigned to the chain operation his contract with the distributor, the chain operation was deemed to have assumed the store owner's duties under that contract as well as being assigned the rights thereunder. Thus, the chain operation can be held to the duty to pay for the health food products that were ordered in April. Although the chain operation is deemed to have assumed the duties of the store owner under the contract, the store owner (the delegator) remains liable on the contract. Consequently, the food research corporation has a right of action for the April payment against both the store owner and the chain operation (although it will be limited to only one recovery, see below). (A) and (C) are incorrect because they conclude that, as between the store owner and the chain operation, only one of them has the duty to tender the required payment. As explained above, both the store owner and the chain operation are under this duty. (D) is incorrect because, as discussed above, both the store owner and the chain operation are liable. Of course, there will be only one recovery, so as not to cause a windfall as a result of the breach. Generally, the purpose of contract damages is to put the nonbreaching party in the position it would have been in had the contract been performed.

A sailing enthusiast went to a boat builder and told him that he wanted a yacht built to his specifications. They agreed that the price would be $400,000, and that the sailing enthusiast was to make payment in full within 30 days after he had accepted delivery of the yacht. They further agreed that the boat builder would not subcontract any of the work. The boat builder, however, contacted a master sail maker and subcontracted the sails for the yacht to him. They agreed orally that the boat builder would pay the sail maker $25,000 for the sails within 20 days of receiving them. The boat builder did not tell the sail maker of his agreement with the sailing enthusiast regarding subcontracting. The sail maker made the sails and delivered them to the boat builder, who then completed the yacht and delivered the boat to the sailing enthusiast. Although the yacht was built to his specifications, the sailing enthusiast refused to accept it after he learned that the boat builder had subcontracted for the sails. When the 20-day payment period for the sails had expired, the sail maker went to the boat builder and demanded the $25,000. The boat builder told the sail maker that he could not pay the $25,000 unless the sailing enthusiast paid him for the yacht. Is there an enforceable contract between the sail maker and the boat builder? A Yes, because they are merchants under the Uniform Commercial Code. B Yes, because the sail maker fully performed. C No, because the boat builder had agreed not to subcontract. D No, because of the Statute of Frauds.

There is an enforceable contract between the sail maker and the boat builder because the sail maker fully performed. The contract here was for the sale of goods (sails) for the price of $500 or more; thus, the contract is within the Statute of Frauds. A contract within the Statute of Frauds is generally unenforceable absent a memorandum signed by the party to be charged containing the contract's essential terms. However, there is an exception to the general rule for goods received and accepted. Here, although the contract was oral, the boat builder accepted the sails, and so he is bound despite the Statute. Note that the boat builder might also be bound under another exception to the Statute - for specially manufactured goods if the sails were made specially for the yacht and were not suitable for sale to others. (A) is incorrect because the U.C.C. does not exempt merchants from the Statute of Frauds. Although it provides a special confirmatory memo rule by which a merchant may be bound even if his signature does not appear on the writing evidencing the contract, here there is no writing at all. (C) is incorrect because it is irrelevant. The fact that the boat builder agreed not to subcontract is relevant to whether he breached his contract with the sailing enthusiast, but it does not affect his contract with the sail maker. (D) is incorrect because, as stated above, the contract here falls within an exception to the Statute of Frauds because the boat builder accepted the sails.

In an installment contract situation, the contract can be canceled by the buyer if: A Any shipment is deficient in quantity B There is a nonconformity in a shipment that substantially impairs the value of the installment and cannot be cured C Any shipment fails to conform to the contract in any way D There is a nonconformity in a shipment that substantially impairs the value of the contract and cannot be cured

To cancel the entire installment contract due to breach, the buyer must show that the nonconformity substantially impairs the value of the entire contract and cannot be cured. If the nonconformity substantially impairs the value of only the installment and cannot be cured, only the single installment may be rejected. Unlike most contracts for the sale of goods, under which a shipment may be rejected in it fails to conform to the contract in any way, installment contracts have special rules limiting the right to reject to substantial impairment of the value of the installment. The right to cancel the entire contract is even further limited to substantial impairment of the value of the entire contract. A shipment that is deficient in quantity is not grounds for canceling an installment contract. This situation is the least likely to even give rise to a right to reject an installment because it is easily cured by a shipment of the missing quantity

The Statute of Frauds requires: A one or more writings that reflect the material terms of the contract, signed by the person sought to be held liable. B a signed writing for suretyship promises that primarily serve the pecuniary interest of the promisor. C the handwritten signature of the party sought to be held liable on some document acknowledging the existence of the contract. D a formal written contract signed by both parties to the agreement.

To satisfy the Statute of Frauds, there must be one or more writings signed by the person sought to be held liable on the contract that reflect the material terms of the contract. The Statute of Frauds does not require a formal written contract signed by both of the parties. For example, a letter, receipt, or a check containing the material terms (e.g., quantity for sale of goods) and signed by the party to be charged satisfies the Statute of Frauds. The needed signature need not be handwritten, but the document or documents must include the material terms of the contract, not just acknowledge the existence of the contract. Generally, the Statute of Frauds requires that suretyship promises be in writing and signed by the party to be held liable. However, there is an exception for suretyship promises that primarily serve the pecuniary interest of the promisor; they are not within the Statute of Frauds.

When two merchants enter into an oral contract for the sale of goods and one party sends to the other party a signed, written confirmation of the agreement, it: A Binds both sender and the recipient, provided the recipient signs the confirmation within 10 days of receipt B Binds both the sender and recipient, provided the recipient had reason to know of its contents and did not object in writing within 10 days of receipt C Binds only the sender because it is signed only by the sender D Binds both the sender and the recipient, provided the recipient actually read the document and did not object in writing within 10 days of receipt

Under the confirmatory memo rule, in contracts between merchants, if one party, within a reasonable time after an oral agreement has been made, sends to the other party a written confirmation of the understanding that is sufficient under law to bind the sender, it will also bind the recipient if he has reason to know of the confirmation's contents and does not object to it in writing within 10 days of receipt. The recipient need not actually read the document; it is binding if the recipient had reason to know of its contents. Even though the memo is generally signed only by the sender, the recipient is bound; the recipient's signature is not necessary.

A vague term in a contract can be cured by: A the presumption that the parties' intent was to include a reasonable term B gap fillers C part performance D quantum meruit

Where part performance supplies the needed clarification of the terms, it can be used to cure vagueness. Gap fillers and the presumption that the parties' intent was to include a reasonable term go to supplying missing, rather than vague, terms. When the parties have included a term that makes the contract too vague to be enforced, the court will not apply a gap-filling term or a presumption to cure the problem. Quantum meruit is another term for quasi-contractual recovery to remedy unjust enrichment. Although it does not cure a vague term, it is available as a remedy for a party who performs despite a vague term that causes a contract to fail.

A builder and a wealthy landowner entered into a written contract whereby the builder would build on the grounds of the landowner's estate a mausoleum, using imported Italian granite, to hold the remains of the landowner's recently deceased wife. The cost of the mausoleum was set at $100,000. After the contract was signed but before construction began, the builder learned that an unforeseen embargo prevented him from getting the granite he planned to use to build the mausoleum. He could get the granite from another source, but it would cost an additional $25,000. The builder explained the situation to the landowner, who agreed to pay $125,000 to have the mausoleum built. The builder prepared a writing stating that the price for the mausoleum was now $125,000. Both the builder and the landowner signed the writing. After the work was completed, the landowner gave the builder a certified check for $100,000 and refused to pay one penny more. If the builder brings suit against the landowner to recover the additional $25,000, will the builder likely prevail? A Yes, because the modification was fair and equitable in view of the unanticipated increase in the cost of granite. B Yes, because the later agreement was in writing and signed by the parties. C No, because the builder had a preexisting duty to do the work for $100,000. D No, because the 25% increase in price that the builder was trying to force on the landowner is unconscionable.

ent (Second), a promise modifying a duty under a contract not fully performed on either side is binding if the modification is fair and equitable in view of circumstances not anticipated by the parties when the contract was made. Here, the contract had not been performed on either side at the time of the modification, the embargo is a circumstance unanticipated by the parties when the contract was made, and the increase is fair and equitable in light of the circumstances (it reflects only the builder's increased costs). Thus, the builder should succeed in his suit for the additional $25,000. (B) is incorrect because whether there is a signed writing is immaterial when there is a failure of consideration and thus no contract to be enforced. (C) is incorrect because it states the traditional rule and does not take into account the unanticipated circumstances of the embargo. Generally, modification of a building contract requires consideration, and performance of a preexisting legal duty is not consideration. Even the slightest change in performance is generally sufficient to constitute consideration, but here there was no change in the builder's duties in exchange for the additional $25,000 payment. Thus, under the traditional view, the modification would not have been valid, and the landowner would have been liable only for the amount agreed to in the original contract.(D) is incorrect because courts rarely recognize unconscionability based on price alone—and then only if the parties were in vastly unequal bargaining positions, which does not appear to be the case here. Moreover, a 25% increase alone is very unlikely to be found unconscionable. Had there been consideration, this modification undoubtedly would have been enforceable.

Which of the following is a true statement regarding discharge of contractual duties by rescission? A An agreement to rescind is itself a binding contract supported by consideration B Once an offeree of a unilateral contract has fully performed, the contract cannot be rescinded under any circumstances C A third-party beneficiary contract may be discharged by mutual rescission despite the vesting of the beneficiary's rights if the beneficiary is a donee beneficiary D A mutual rescission agreement must be in writing if the contract so provides

"An agreement to rescind is itself a binding contract supported by consideration" is a true statement. The consideration is the giving up by each party of her right to counterperformance from the other. A third-party beneficiary contract may not be discharged by mutual rescission if the third-party beneficiary's rights have vested, regardless of whether he is a creditor or donee beneficiary. Once an offeree of a unilateral contract has fully performed, the contract can be rescinded, but only if the rescission promise is supported by: (i) An offer of new consideration by the nonperforming party; (ii) Elements of promissory estoppel; or (iii) Manifestation of an intent by the offeree to make a gift of the obligation owed to her. A mutual rescission agreement may be oral, even if the contract to be rescinded expressly states that it can be rescinded only by a writing.


Kaugnay na mga set ng pag-aaral

Chapter 13: inventory management with perishable demand

View Set